You are on page 1of 121

MATHEMATICS CONTESTS

THE AUSTRALIAN SCENE 2015


PART 2: INVITATIONAL COMPETITIONS
A Di Pasquale and N Do

Le ar n f r om ye ste r d a y, l ive f o r to day, ho pe f o r to m o r r ow.


T h e i m p or ta nt th i ng is no t to sto p que stio ning.
Alber t Einstein

A u s t r a l i a n M a t h e ma t i c a l O l y m p i a d C omm i t t e e

A department of the A u s t r a l i a n M at h e mat i c s T r u s t

1
Published by

AMT Publishing

Australian Mathematics Trust


University of Canberra Locked Bag 1
Canberra GPO ACT 2601
Australia

Tel: 61 2 6201 5137

www.amt.edu.au

AMTT Limited

ACN 083 950 341

Copyright 2015 by the Australian Mathematics Trust

National Library of Australia Card Number

and ISSN 1323-6490

2
SUPPORT FOR THE AUSTRALIAN MATHEMATICAL
OLYMPIAD COMMITTEE TRAINING PROGRAM

The Australian Mathematical Olympiad Committee Training Program is an activity of the Australian Mathematical
Olympiad Committee, a department of the Australian Mathematics Trust.

Trustee
The University of Canberra

Sponsors
The Mathematics/Informatics Olympiads are supported by the Australian Government Department of Education
and Training through the Mathematics and Science Participation Program.
The Australian Mathematical Olympiad Committee (AMOC) also acknowledges the significant financial support
it has received from the Australian Government towards the training of our Olympiad candidates and the
participation of our team at the International Mathematical Olympiad (IMO).
The views expressed here are those of the authors and do not necessarily represent the views of the
government.

Special Thanks
With special thanks to the Australian Mathematical Society, the Australian Association of Mathematics Teachers
and all those schools, societies, families and friends who have contributed to the expense of sending the 2015
IMO team to Chiang Mai, Thailand.

3
ACKNOWLEDGEMENTS

The Australian Mathematical Olympiad Committee thanks sincerely all sponsors, teachers, mathematicians and
others who have contributed in one way or another to the continued success of its activities.
The editors thank sincerely those who have assisted in the compilation of this book, in particular the students
who have provided solutions to the 2015 IMO. Thanks also to members of AMOC and Challenge Problems
Committees, Adjunct Professor Mike Clapper, staff of the Australian Mathematics Trust and others who are
acknowledged elsewhere in the book.

4
PREFACE

After last year, there seemed little room for improvement, but 2015 has
been even better, marked particularly by our best ever result at an IMO,
where we were placed 6th out of the 104 competing countries, finishing
ahead of all European countries (including Russia) and many other
traditional powerhouses such as Singapore, Japan and Canada. For the
first time ever, all six team members obtained Silver or better, with two
team members (Alex Gunning and Seyoon Ragavan) claiming Gold.
Alex finished fourth in the world (after his equal first place last year, and
becomes Australias first triple Gold medallist in any academic Olympiad.
Seyoon, who finished 19th, now has three IMOs under his belt with a year
still to go. Once again, we had three Year 12 students in the team, so there
will certainly be opportunities for new team members next year. It was also
pleasing to, once again, see an Australian-authored question on the paper,
this year, the prestigious Question 6, which was devised by Ivan Guo and
Ross Atkins, based on the mathematics of juggling.
In the Mathematics Ashes we tied with the British team; however, we
finished comfortably ahead of them in the IMO competition proper. Director of Training and IMO Team Leader,
Dr Angelo Di Pasquale, along with his Deputy Andrew Elvey Price and their team of former Olympians continue
to innovate and keep the training alive, fresh and, above all, of high quality. The policy of tackling very hard
questions in training was daunting for team members at times but seems to have paid off.
The Mathematics Challenge for Young Australians (MCYA) also continues to attract strong entries, with the
Challenge continuing to grow, helped by the gathering momentum of the new Middle Primary Division, which
began in 2014. The Enrichment stage, containing course work, allows students to broaden their knowledge
base in the areas of mathematics associated with the Olympiad programs and more advanced problem-solving
techniques. We have continued running workshops for teachers to develop confidence in managing these
programs for their more able studentsthis seems to be paying off with strong numbers in both the Challenge
and Enrichment stages. The final stage of the MCYA program is the Australian Intermediate Mathematics
Olympiad (AIMO). It is a delight to record that over the last two years the number of entries to AIMO has
doubled. This has been partly due to wider promotion of the competition, but more specifically a result of
the policy of offering free entry to AMC prize winners. There were some concerns in 2014 that some of the
new contestants were under-prepared for AIMO and there were more zero scores than we would have liked.
However, this year, the number of zero scores was less than 1% and the quality of papers was much higher,
revealing some significant new talent, some of whom will be rewarded with an invitation to the December School
of Excellence.
There are many people who contribute to the success of the AMOC program. These include the Director of
Training and the ex-Olympians who train the students at camps; the AMOC State Directors; and the Challenge
Director, Dr Kevin McAvaney, and the various members of his Problems Committee, who develop such original
problems, solutions and discussions each year. The AMOC Senior Problems Committee is also a major
contributor and Norman Do is continuing with his good work. The invitational program saw some outstanding
results from Australian students, with a number of perfect scores. Details of these achievements are provided in
the appropriate section of this book. As was the case last year, we are producing Mathematics ContestsThe
Australian Scene in electronic form only and making it freely available through the website. We hope this will
provide greater access to the problems and section reports. This book is also available in two sections, one
containing the MCYA reports and papers and the other containing the Olympiad training program reports and
papers.

Mike Clapper
November 2015

5
CONTENTS

Support for the Australian Mathematical Olympiad committee Training Program 3

Acknowledgements 4

Preface 5

Contents 6

Background Notes on the IMO and AMOC 7

Membership of AMOC Committees 10

AMOC Timetable for Selection of the Team to the 2016 IMO 11

Activities of AMOC Senior Problems Committee 12

AMOC Senior Contest 13

AMOC Senior Contest Solutions 14

AMOC Senior Contest Results 22

AMOC Senior Contest Statistics 23

AMOC School of Excellence 24

Australian Mathematical Olympiad 26

Australian Mathematical Olympiad Solutions 28

Australian Mathematical Olympiad Statistics 53

Australian Mathematical Olympiad Results 54

27th Asian Pacific Mathematics Olympiad 56

27th Asian Pacific Mathematics Olympiad Solutions 57

27th Asian Pacific Mathematics Olympiad Results 71

AMOC Selection School 73

IMO Team Preparation School 76

The Mathematics Ashes 77

The Mathematics Ashes Results 78

IMO Team Leaders Report 79

International Mathematical Olympiad 83

International Mathematical Olympiad Solutions 85

International Mathematical Olympiad Results 111

Origin of Some Questions 116

Honour Roll 117

6
BACKGROUND NOTES ON THE IMO AND AMOC

The Australian Mathematical Olympiad Committee


In 1980, a group of distinguished mathematicians formed the Australian Mathematical Olympiad Committee
(AMOC) to coordinate an Australian entry in the International Mathematical Olympiad (IMO).
Since then, AMOC has developed a comprehensive program to enable all students (not only the few who aspire
to national selection) to enrich and extend their knowledge of mathematics. The activities in this program are not
designed to accelerate students. Rather, the aim is to enable students to broaden their mathematical experience
and knowledge.
The largest of these activities is the MCYA Challenge, a problem-solving event held in second term, in which
thousands of young Australians explore carefully developed mathematical problems. Students who wish to
continue to extend their mathematical experience can then participate in the MCYA Enrichment Stage and pursue
further activities leading to the Australian Mathematical Olympiad and international events.
Originally AMOC was a subcommittee of the Australian Academy of Science. In 1992 it collaborated with the
Australian Mathematics Foundation (which organises the Australian Mathematics Competition) to form the
Australian Mathematics Trust. The Trust, a not-for-profit organisation under the trusteeship of the University
of Canberra, is governed by a Board which includes representatives from the Australian Academy of Science,
Australian Association of Mathematics Teachers and the Australian Mathematical Society.
The aims of AMOC include:
(1) giving leadership in developing sound mathematics programs in Australian schools
(2) identifying, challenging and motivating highly gifted young Australian school students in mathematics
(3) training and sending Australian teams to future International Mathematical Olympiads.

AMOC schedule from August until July for potential IMO team members
Each year hundreds of gifted young Australian school students are identified using the results from the
Australian Mathematics Competition sponsored by the Commonwealth Bank, the Mathematics Challenge for
Young Australians program and other smaller mathematics competitions. A network of dedicated mathematicians
and teachers has been organised to give these students support during the year either by correspondence sets
of problems and their solutions or by special teaching sessions.
It is these students who sit the Australian Intermediate Mathematics Olympiad, or who are invited to sit the
AMOC Senior Contest each August. Most states run extension or correspondence programs for talented
students who are invited to participate in the relevant programs. The 25 outstanding students in recent AMOC
programs and other mathematical competitions are identified and invited to attend the residential AMOC School
of Excellence held in December.
In February approximately 100 students are invited to attempt the Australian Mathematical Olympiad. The
best 20 or so of these students are then invited to represent Australia in the correspondence Asian Pacific
Mathematics Olympiad in March. About 12 students are selected for the AMOC Selection School in April and
about 13 younger students are also invited to this residential school. Here, the Australian team of six students
plus one reserve for the International Mathematical Olympiad, held in July each year, is selected. A personalised
support system for the Australian team operates during May and June.
It should be appreciated that the AMOC program is not meant to develop only future mathematicians.
Experience has shown that many talented students of mathematics choose careers in engineering, computing,
and the physical and life sciences, while others will study law or go into the business world. It is hoped that the
AMOC Mathematics Problem-Solving Program will help the students to think logically, creatively, deeply and with
dedication and perseverance; that it will prepare these talented students to be future leaders of Australia.

The International Mathematical Olympiad


The IMO is the pinnacle of excellence and achievement for school students of mathematics throughout the
world. The concept of national mathematics competitions started with the Etvos Competition in Hungary during
1894. This idea was later extended to an international mathematics competition in 1959 when the first IMO was

7
held in Romania. The aims of the IMO include:
(1) discovering, encouraging and challenging mathematically gifted school students
(2) fostering friendly international relations between students and their teachers
(3) sharing information on educational syllabi and practice throughout the world.
It was not until the mid-sixties that countries from the western world competed at the IMO. The United States of
America first entered in 1975. Australia has entered teams since 1981.
Students must be under 20 years of age at the time of the IMO and have not enrolled at a tertiary institution.
The Olympiad contest consists of two four-and-a-half hour papers, each with three questions.
Australia has achieved varying successes as the following summary of results indicate. HM (Honorable Mention)
is awarded for obtaining full marks in at least one question.
The IMO will be held in Hong Kong in 2016.

8
Summary of Australias achievements at previous IMOs

Year City Gold Silver Bronze HM Rank


1981 Washington 1 23 out of 27 teams
1982 Budapest 1 21 out of 30 teams
1983 Paris 1 2 19 out of 32 teams
1984 Prague 1 2 15 out of 34 teams
1985 Helsinki 1 1 2 11 out of 38 teams
1986 Warsaw 5 15 out of 37 teams
1987 Havana 3 15 out of 42 teams
1988 Canberra 1 1 1 17 out of 49 teams
1989 Braunschweig 2 2 22 out of 50 teams
1990 Beijing 2 4 15 out of 54 teams
1991 Sigtuna 3 2 20 out of 56 teams
1992 Moscow 1 1 2 1 19 out of 56 teams
1993 Istanbul 1 2 3 13 out of 73 teams
1994 Hong Kong 2 3 3 12 out of 69 teams
1995 Toronto 1 4 1 21 out of 73 teams
1996 Mumbai 2 3 23 out of 75 teams
1997 Mar del Plata 2 3 1 9 out of 82 teams
1998 Taipei 4 2 13 out of 76 teams
1999 Bucharest 1 1 3 1 15 out of 81 teams
2000 Taejon 1 3 1 16 out of 82 teams
2001 Washington D.C. 1 4 25 out of 83 teams
2002 Glasgow 1 2 1 1 26 out of 84 teams
2003 Tokyo 2 2 2 26 out of 82 teams
2004 Athens 1 1 2 1 27 out of 85 teams
2005 Merida 6 25 out of 91 teams
2006 Ljubljana 3 2 1 26 out of 90 teams
2007 Hanoi 1 4 1 22 out of 93 teams
2008 Madrid 5 1 19 out of 97 teams
2009 Bremen 2 1 2 1 23 out of 104 teams
2010 Astana 1 3 1 1 15 out of 96 teams
2011 Amsterdam 3 3 25 out of 101 teams
2012 Mar del Plata 2 4 27 out of 100 teams
2013 Santa Marta 1 2 3 15 out of 97 teams
1
Perfect Score
2014 Cape Town 3 2 11 out of 101 teams
by Alexander
Gunning
2015 Chiang Mai 2 4 6 out of 104 teams

9
MEMBERSHIP OF AMOC COMMITTEES

Australian Mathematical Olympiad Committee 2015


Chair
Prof C Praeger, University of Western AustraliaDeputy Chair
Assoc Prof DHunt, University of New South Wales

Executive Director
Adj Prof Mike Clapper, Australian Mathematics Trust, ACT

Treasurer
Dr P Swedosh, The King David School, VIC

Chair, Senior Problems Committee


Dr N Do, Monash University, VIC

Chair, Challenge
Dr K McAvaney, Deakin University, VIC

Director of Training and IMO Team Leader


Dr A Di Pasquale, University of Melbourne, VIC

IMO Deputy Team Leader


Mr A Elvey Price, University of Melbourne, VIC

State Directors
Dr K Dharmadasa, University of Tasmania
Dr G Gamble, University of Western Australia
Dr Ian Roberts, Northern Territory
Dr W Palmer, University of Sydney, NSW
Mr D Martin, South Australia
Dr V Scharaschkin, University of Queensland
Dr P Swedosh, The King David School, VIC
Dr Chris Wetherell, Radford College, ACT

Representatives
Ms A Nakos, Challenge Committee
Prof M Newman, Challenge Committee
Mr H Reeves, Challenge Committee

10
AMOC TIMETABLE FOR SELECTION OF THE TEAM TO THE
2016 IMO
August 2015July 2016
Hundreds of students are involved in the AMOC programs which begin on a state basis. The students are given
problem-solving experience and notes on various IMO topics not normally taught in schools.
The students proceed through various programs with the top 25 students, including potential team members and
other identified students, participating in a ten-day residential school in December.
The selection program culminates with the April Selection School during which the team is selected.
Team members then receive individual coaching by mentors prior to assembling for last minute training before
the IMO.

Month Activity
Outstanding students are identified from AMC results, MCYA, other competitions
and recommendations; and eligible students from previous training programs
August AMOC state organisers invite students to participate in AMOC programs
Various state-based programs
AMOC Senior Contest

September Australian Intermediate Mathematics Olympiad

December AMOC School of Excellence

Summer Correspondence Program for those who attended the School of


January
Excellence

February Australian Mathematical Olympiad

March Asian Pacific Mathematics Olympiad

April AMOC Selection School

MayJune Personal Tutor Scheme for IMO team members

Short mathematics school for IMO team members


July
2016 IMO in Hong Kong.

11
ACTIVITIES OF AMOC SENIOR PROBLEMS COMMITTEE

This committee has been in existence for many years and carries out a number of roles. A central role is the
collection and moderation of problems for senior and exceptionally gifted intermediate and junior secondary school
students. Each year the Problems Committee provides examination papers for the AMOC Senior Contest and the
Australian Mathematical Olympiad. In addition, problems are submitted for consideration to the Problem Selection
Committees of the annual Asian Pacific Mathematics Olympiad and the International Mathematical Olympiad.

AMOC Senior Problems Committee October 2014September 2015


Dr A Di Pasquale, University of Melbourne, VIC
Dr N Do, Monash University, VIC, (Chair)
Dr M Evans, Australian Mathematical Sciences Institute, VIC
Dr I Guo, University of Sydney, NSW
Assoc Prof D Hunt, University of NSW
Dr J Kupka, Monash University, VIC
Assoc Prof H Lausch, Monash University, VIC
Dr K McAvaney, Deakin University, VIC
Dr D Mathews, Monash University, VIC
Dr A Offer, Queensland
Dr C Rao, NEC Australia, VIC
Dr B B Saad, Monash University, VIC
Assoc Prof J Simpson, Curtin University of Technology, WA
Emeritus Professor P J Taylor, Australian Capital Territory
Dr I Wanless, Monash University, VIC

1. 2015 Australian Mathematical Olympiad


The Australian Mathematical Olympiad (AMO) consists of two papers of four questions each and was sat on
10 and 11 February. There were 106 participants including 11 from New Zealand, seven more participants than
2014. Two students, Alexander Gunning and Seyoon Ragavan, achieved perfect scores and nine other students
were awarded Gold certificates,15 students were awarded Silver certificates and 26 students were awarded
Bronze certificates.

2. 2015 Asian Pacific Mathematics Olympiad


On Tuesday 10 March students from nations around the Asia-Pacific region were invited to write the Asian
Pacific Mathematics Olympiad (APMO). Of the top ten Australian students who participated, there were 1 Gold,
2 Silver, 4 Bronze and 3 HM certificates awarded. Australia finished in 9th place overall.

3. 2015 International Mathematical Olympiad, Chiang Mai, Thailand.


The IMO consists of two papers of three questions worth seven points each. They were attempted by teams of
six students from 104 countries on 8 and 9 July in Cape Town, South Africa. Australia was placed 6th of 104
countries, its most successful result since it began participating. The medals for Australia were two Gold and
four Silver.

4. 2015 AMOC Senior Contest


Held on Tuesday 11 August, the Senior Contest was sat by 84 students (compared to 81 in 2014). There
were three students who obtained perfect scores and two other students who were also Prize winners. Three
studentent obtained High Distinctions and 14 students obtained Distinctions.

12
THE 2015 AMOC
AMOC SENIOR
SENIOR CONTEST
CONTEST
Tuesday, 11 August 2015
Time allowed: 4 hours
No calculators are to be used.
Each question is worth seven points.

1. A number is called k-addy if it can be written as the sum of k consecutive positive


integers. For example, the number 9 is 2-addy because 9 = 4 + 5 and it is also 3-addy
because 9 = 2 + 3 + 4.

(a) How many numbers in the set {1, 2, 3, . . . , 2015} are simultaneously 3-addy,
4-addy and 5-addy?
(b) Are there any positive integers that are simultaneously 3-addy, 4-addy, 5-addy
and 6-addy?

2. Consider the sequence a1 , a2 , a3 , . . . defined by a1 = 1 and


1a1 + 2a2 + 3a3 + + mam
am+1 = for m 1.
am
Determine the largest integer n such that an < 1 000 000.

3. A group of students entered a mathematics competition consisting of five problems.


Each student solved at least two problems and no student solved all five problems.
For each pair of problems, exactly two students solved them both.
Determine the minimum possible number of students in the group.

4. Let ABCD be a rectangle with AB > BC. Let E be the point on the diagonal AC
such that BE is perpendicular to AC. Let the circle through A and E whose centre
lies on the line AD meet the side CD at F .
Prove that BF bisects the angle AF C.

5. For a real number x, let x be the largest integer less than or equal to x.
Find all prime numbers p for which there exists an integer a such that
       
a 2a 3a pa
+ + + + = 100.
p p p p

c 2015 Australian Mathematics Trust




13
THE 2015 AMOC SENIOR CONTEST
Solutions and cumulative marking scheme
AMOC
c 2015 Australian
 SENIOR
Mathematics Trust CONTEST SOLUTIONS

1. A number is called k-addy if it can be written as the sum of k consecutive positive integers.
For example, the number 9 is 2-addy because 9 = 4 + 5 and it is also 3-addy because
9 = 2 + 3 + 4.

(a) How many numbers in the set {1, 2, 3, . . . , 2015} are simultaneously 3-addy, 4-addy
and 5-addy?
(b) Are there any positive integers that are simultaneously 3-addy, 4-addy, 5-addy and
6-addy?

Solution (Angelo Di Pasquale)


Since (a 1) + a + (a + 1) = 3a, the 3-addy numbers are precisely those that are divisible
by 3 and greater than 3. 1
Since (a 2) + (a 1) + a + (a + 1) + (a + 2) = 5a, the 5-addy numbers are precisely those
that are divisible by 5 and greater than 10.
Since (a 1) + a + (a + 1) + (a + 2) = 4a + 2, the 4-addy numbers are precisely those that
are congruent to 2 modulo 4 and greater than 6. 2

(a) From the observations above, a positive integer is simultaneously 3-addy, 4-addy and
5-addy if and only if it is divisible by 3, divisible by 5, divisible by 2, and not divisible
by 4. Such numbers are of the form 30m, where m is a positive odd integer. 3
Since 67 30 = 2010, the number of elements of the given set that are simultaneously
3-addy, 4-addy and 5-addy is 68
2 = 34. 4
(b) Since (a 2) + (a 1) + a + (a + 1) + (a + 2) + (a + 3) = 6a + 3, all 6-addy numbers
are necessarily odd. 5
On the other hand, we have already deduced that all 4-addy numbers are even. 6
Therefore, there are no numbers that are simultaneously 4-addy and 6-addy. 7

14
2. Consider the sequence a1 , a2 , a3 , . . . defined by a1 = 1 and
1a1 + 2a2 + 3a3 + + mam
am+1 = for m 1.
am
Determine the largest integer n such that an < 1 000 000.

Solution 1 (Norman Do)


First, we note that all terms of the sequence are positive rational numbers. Below, we
rewrite the defining equation for the sequence in both its original form and with the value
of m shifted by 1.

am am+1 = 1a1 + 2a2 + 3a3 + + mam


am+1 am+2 = 1a1 + 2a2 + 3a3 + + mam + (m + 1)am+1 1

Subtracting the first equation from the second yields

am+1 am+2 am am+1 = (m + 1)am+1 am+2 am = m + 1,

for all m 1. 2
So for m = 2k + 1 an odd positive integer,

a2k+1 a1 = (a2k+1 a2k1 ) + (a2k1 a2k3 ) + + (a3 a1 )


= 2k + (2k 2) + + 2
= 2 [k + (k 1) + + 1]
= k(k + 1). 3

Similarly, for m = 2k an even positive integer,

a2k a2 = (a2k a2k2 ) + (a2k2 a2k4 ) + + (a4 a2 )


= (2k 1) + (2k 3) + + 3
= k 2 1. 4

Using a1 = 1 and a2 = 1, we obtain the formula



m2 , if m is even,
4
am = 5
m +3 , if m is odd.
2
4

If m is odd, then
(m + 1)2 m2 + 3 2m 2
am+1 am = = ,
4 4 4
and if m is even, then

(m + 1)2 + 3 m2 2m + 4
am+1 am = = . 6
4 4 4
In particular, it follows that a2 < a3 < a4 < . Since a2000 = 1 000 000, the largest
integer n such that an < 1 000 000 is 1999. 7

15
Solution 2 (Angelo Di Pasquale)
We will prove by induction that a2k1 = k 2 k + 1 and a2k = k 2 for each positive integer
k. The base case k = 1 is true since a1 = a2 = 1. Now assume that the two formulas hold
for k = 1, 2, . . . , n. We will show that they also hold for k = n + 1. 2
Consider the following sequence of equalities.
2n
 n
 n

iai = (2i 1) a2i1 + 2i a2i
i=1 i=1 i=1
n n

= (2i 1) (i2 i + 1) + (2i) (i2 ) (by the inductive assumption)
i=1 i=1
n

= 4i3 3i2 + 3i 1
i=1
n

= 3i3 + (i 1)3 3
i=1
n
 n1

=3 i3 + i3
i=1 i=1
3n2 (n + 1)2 (n 1)2 n2  n
 n2 (n + 1)2 
= + since i3 =
4 4 4
i=1
2 2
= n (n + n + 1)
= (n2 + n + 1)a2n (by the inductive assumption)

It follows that
1a1 + 2a2 + 3a3 + + 2na2n
a2n+1 = = n2 + n + 1. 4
a2n

Now consider the following sequence of equalities, which uses the facts derived above that

state that 2n 2 2 2
i=1 iai = n (n + n + 1) and a2n+1 = n + n + 1.

2n+1

iai = n2 (n2 + n + 1) + (2n + 1)a2n+1
i=1

= n2 a2n+1 + (2n + 1)a2n+1


= (n + 1)2 a2n+1

It follows that
1a1 + 2a2 + 3a3 + + (2n + 1)a2n+1
a2n+2 = = (n + 1)2 . 5
a2n+1

So we have shown that the two formulas a2k1 = k 2 k + 1 and a2k = k 2 hold for
k = 1, 2, . . . , n + 1. This completes the induction and the rest of the proof follows Solution
1. 7

16
3. A group of students entered a mathematics competition consisting of five problems. Each
student solved at least two problems and no student solved all five problems. For each
pair of problems, exactly two students solved them both.
Determine the minimum possible number of students in the group.

Solution (Norman Do)


It is possible that the group comprised six students, as demonstrated by the following
example.

Student 1 solved problems 1, 2, 3, 4. Student 4 solved problems 2, 4, 5.


Student 2 solved problems 1, 2, 3, 5. Student 5 solved problems 3 and 4.
Student 3 solved problems 1, 4, 5. Student 6 solved problems 3 and 5. 2

Suppose that a students solved 4 problems, b students solved 3 problems, and c students

solved 2 problems. Therefore, a students solved 42 = 6 pairs of problems, b students
 
solved 32 = 3 pairs of problems and c students solved 22 = 1 pair of problems. Since we
have shown an example in which the number of students in the group is 6, let us assume
that a + b + c 5.

There are 52 = 10 pairs of problems altogether and, for each pair of problems, exactly
two students solved them both, so we must have

6a + 3b + c = 20. 4

Reading the above equation modulo 3 yields c 2 (mod 3). If c 5, then we have
a + b + c 6, contradicting our assumption. Therefore, we must have c = 2 and 2a + b = 6.
For a + b + c 5, the only solution is given by (a, b, c) = (3, 0, 2). 5
However, it is impossible for 3 students to have solved 4 problems each. That would mean
that each of the 3 students did not solve exactly 1 problem. So there would exist a pair of
problems for which 3 students solved them both, contradicting the required conditions.
In conclusion, the minimum possible number of students in the group is 6. 7

17
4. Let ABCD be a rectangle with AB > BC. Let E be the point on the diagonal AC such
that BE is perpendicular to AC. Let the circle through A and E whose centre lies on the
line AD meet the side CD at F .
Prove that BF bisects the angle AF C.

Solution 1 (Alan Offer)


Let the circle through A and E whose centre lies on AD meet the line AD again at H.

A B

D C
F

Since F D is the altitude of the right-angled triangle AF H, we have AF H ADF .


Since triangles AEH and ADC are right-angled with a common angle at A, we have
AEH ADC. Since BE is the altitude of the right-angled triangle ABC, we have
ABC AEB. These three pairs of similar triangles lead respectively to the three
pairs of equal ratios
AF AH AE AH AB AC
= = = . 3
AD AF AD AC AE AB

Putting these together, we have

AF 2 = AD AH = AC AE = AB 2 . 6

So triangle BAF is isosceles and we have

AF B = ABF = 90 CBF = CF B,

where the last equality uses the angle sum in triangle BCF . Since AF B = CF B, we
have proven that BF bisects the angle AF C. 7

Solution 2 (Angelo Di Pasquale)


First, note that the circumcircles of triangle AEF and triangle BEC are tangent to the
line AB at A and B, respectively. Considering the power of the point A with respect to
the circumcircle of triangle BEC, we have

AB 2 = AE AC. 1

18
Next, by the alternate segment theorem and the fact that AB  CD, we have EF A =
EAB = ECF . Hence, by the alternate segment theorem again, the circumcircle of
triangle EF C is tangent to the line AF at F . Considering the power of the point A with
respect to the circumcircle of triangle EF C, we have

AF 2 = AE AC. 4

Comparing the two equations above, we deduce that AB = AF . 6


Hence, AF B = ABF = CF B, as required. 7

Solution 3 (Ivan Guo)


Let the circle through A and E whose centre lies on AD meet the line AD again at H.
Then AEB = AEH = 90 , so the points B, E and H are collinear. Consider the
inversion f with centre A and radius AF . 2
Note that the circumcircle of the cyclic quadrilateral AEF H must map to a line parallel
to AB through F . Thus,
f (circle AEF H) = line CD. 4

This immediately gives f (C) = E and f (H) = D. Now the circumcircle of ABCD must
map to a line passing through f (C) = E and f (D) = H. This implies that f (B) = B.
Hence, AB = AF . 6
Therefore, we have BF C = ABF = AF B. 7

Solution 4 (Chaitanya Rao)


Consider the following chain of equalities.

DF 2 = AD DH (ADF F DH) 1
= AD (AH AD)
 
AB 2
= AD AD (ABC HAB) 3
BC
= AB 2 AD2 (AD = BC) 4

Hence, AB 2 = DF 2 + AD2 = AF 2 , which implies that AB = AF . 6


So triangle BAF is isosceles and we have

AF B = ABF = 90 CBF = CF B,

where the last equality uses the angle sum in triangle BCF . Since AF B = CF B, we
have proven that BF bisects the angle AF C. 7

19
5. For a real number x, let x be the largest integer less than or equal to x.
Find all prime numbers p for which there exists an integer a such that
       
a 2a 3a pa
+ + + + = 100.
p p p p

Solution 1 (Norman Do)


The possible values for p are 2, 5, 17 and 197.
We divide the problem into the following two cases.

The number a is divisible by p.


If we write a = kp, the equation becomes
a(p + 1)
= 100 kp(p + 1) = 200.
2
So both p and p + 1 are positive divisors of 200. However, one can easily see that
there are no such primes p. 1
The number a is not divisible by p.
For a real number x, let {x} = x x. Then we may write the equation as
         
a 2a 3a pa a 2a 3a pa
+ + + + + + + + = 100.
p p p p p p p p
Summing the terms of the arithmetic progression on the left-hand side yields
       
a(p + 1) a 2a 3a pa
+ + + + = 100. 3
2 p p p p
     3a   pa 
We will prove that the sequence of numbers ap , 2a p , p ,..., p is a rearrange-
0 1 2 p1
ment of the sequence of numbers p , p , p , . . . , p . 4
 ka  0 1 2 p1
Observe that if k is a positive integer, then p is one of the numbers p , p , p , . . . , p .
     3a   
So it suffices to show that no two of the numbers ap , 2a p , p , . . . , pa
p are equal.
 ia   ja 
Suppose for the sake of contradiction that p = p , where 1 i < j p. Then
ja a(ji)
p ia
p = p must be an integer. It follows that either a is divisible by p or j i is
divisible by p. However, since we have assumed that a is not divisible by p and that
1 i < j p, we obtain the desired contradiction. Hence, we may conclude that the
     3a   pa 
sequence of numbers ap , 2a p , p ,..., p is a rearrangement of the sequence
0 1 2 p1
of numbers p , p , p , . . . , p . 5
We may now write the equation as
a(p + 1) p 1
= 100 (a 1)(p + 1) = 198. 6
2 2
Therefore, p + 1 is a positive divisor of 198 in other words, one of the numbers

1, 2, 3, 6, 9, 11, 18, 22, 33, 66, 99, 198.

Since p is a prime, it follows that p must be equal to 2, 5, 17 or 197. This leads to


the possible solutions (p, a) = (2, 67), (5, 34), (17, 12), (197, 2). All four of these pairs
satisfy the given equation with a not divisible by p, so we obtain p = 2, 5, 17, 197. 7

20
Solution 2 (Ivan Guo, Angelo Di Pasquale and Ian Wanless)
The possible values for p are 2, 5, 17 and 197.
The case where a is divisible by p is handled in the same way as Solution 1. 1
Furthermore, one can check that the pair (p, a) = (2, 67) satisfies the conditions of the
problem. So assume that p is an odd prime and that a is not divisible by p.
For any integers 1 r, s p 1 with r + s = p, we have p  ra and p  sa. Therefore,
       
ar as ar as ar as ar as
1+ 1< + < + a2< + < a.
p p p p p p p p
   
But since arp + p
as
is an integer, we conclude that
   
ar as
+ = a 1. 3
p p

p1
The numbers {1, 2, . . . , p 1} can be partitioned into 2 pairs whose sum is p. 4
Using the equation above for each such pair and substituting into the original equation,
we obtain  
p1
(a 1) + a = 100 (a 1)(p + 1) = 198. 6
2

Therefore, p + 1 is a positive divisor of 198 in other words, one of the numbers

1, 2, 3, 6, 9, 11, 18, 22, 33, 66, 99, 198.

Since p is a prime, it follows that p must be equal to 2, 5, 17 or 197. This leads to the
possible solutions (p, a) = (2, 67), (5, 34), (17, 12), (197, 2). All four of these pairs satisfy
the given equation with a not divisible by p, so we obtain p = 2, 5, 17, 197. 7

21
AMOC SENIOR CONTEST RESULTS

Name School Year Score

Prize

Yong See Foo Nossal High School VIC 11 35

Kevin Xian James Ruse Agricultural High School NSW 11 35

Wilson Zhipu Zhao Killara High School NSW 11 35

Ilia Kucherov Westall Secondary College VIC 11 34

Seyoon Ragavan Knox Grammar School NSW 11 34

High Distinction

Jongmin Lim Killara High School NSW 11 32

Matthew Cheah Penleigh and Essendon Grammar School VIC 10 29

Jerry Mao Caulfield Grammar School (Wheelers Hill) VIC 9 29

Distinction

Alexander Barber Scotch College VIC 11 28

Michelle Chen Methodist Ladies College VIC 11 28

Thomas Baker Scotch College VIC 11 27

Linus Cooper James Ruse Agricultural High School NSW 9 27

Steven Lim Hurlstone Agricultural High School NSW 9 27

Eric Sheng Newington College NSW 11 27

William Song Scotch College VIC 11 27

Jack Liu Brighton Grammar VIC 9 26

Leo Li Christ Church Grammar School WA 11 25

Bobby Dey James Ruse Agricultural High School NSW 10 24

Michael Robertson Dickson College ACT 11 24

Charles Li Camberwell Grammar School VIC 9 22

Isabel Longbottom Rossmoyne Senior High School WA 10 22

Guowen Zhang St Josephs College, Gregory Terrace QLD 9 22

22
AMOC SENIOR CONTEST STATISTICS

Score Distribution/Problem

Problem Number of Students/Score


Mean
Number 0 1 2 3 4 5 6 7

1 1 0 0 0 1 5 46 31 6.2

2 16 9 8 1 2 8 15 25 4.1

3 25 4 11 2 4 5 8 25 3.5

4 54 13 2 0 0 0 0 15 1.5

5 65 2 1 0 0 4 6 6 1.2

23
2014 AMOCAMOC SCHOOL
School of OF EXCELLENCE
Excellence
The 2014 AMOC School of Excellence was held 110 December at Newman College,
University of Melbourne. The main qualifying exams to be invited to this are the AIMO
and the AMOC Senior Contest.
This year AMOC went ahead with a new initiative in an attempt to widen the net of
identification. In particular, any prize winner in the Australian Mathematics Competition
(AMC) would be given free entry into the AIMO. In this way we hoped to identify top
students from among schools that may not normally enter students in the AIMO. This
turned out to be quite successful and prompted us to increase the number of invitations
we normally make. Consequently, 28 students from around Australia attended the school.
A further student from New Zealand also attended.
The students are divided into a senior group and a junior group. There were 17 junior
students, 16 of whom were attending for the first time. There were 12 students making
up the senior group.
The program covered the four major areas of number theory, geometry, combinatorics
and algebra. Each day would start at 8am with lectures or an exam and go until 12
noon or 1pm. After a one-hour lunch break they would have a lecture at 2pm. At
4pm participants would usually have free time, followed by dinner at 6pm. Finally, each
evening would round out with a problem session, topic review, or exam review from 7pm
until 9pm.
Another new initiative we tried was to invite two of our more experienced senior students
to give a lecture. The rationale behind this is that teaching a subject is highly bene-
ficial to the teacher because it can really solidify the foundations of the teachers own
understanding. A further fringe benefit is that they also get to hone their LATEX skills.
Alexander Gunning was assigned the senior inequalities lecture, and Seyoon Ragavan was
assigned the senior transformation geometry lecture. I did some trial runs with them
prior to the lectures so as to trouble shoot any problems, as well as to give them some
practice for the real thing. Overall it was successful, and I will likely try it again in the
future.
Many thanks to Andrew Elvey Price, Ivan Guo, Victor Khou, and Sampson Wong, who
served as live-in staff. Also my thanks go to Adrian Agisilaou, Norman Do, Patrick He,
Alfred Liang, Daniel Mathews, Konrad Pilch, Chaitanya Rao, and Mel Shu who assisted
in lecturing and marking.

Angelo Di Pasquale
Director of Training, AMOC

24
Participants at the 2014 AMOC School of Excellence

Name School Year

Seniors
Thomas Baker Scotch College VIC 10
Matthew Cheah Penleigh and Essendon Grammar School VIC 9
Yong See Foo Nossal High School VIC 10
Alexander Gunning Glen Waverley Secondary College VIC 11
Leo Li Christ Church Grammar School WA 10
Allen Lu Sydney Grammar School NSW 11
Seyoon Ragavan Knox Grammar School NSW 10
Kevin Shen St Kentigern College NZ 11*
Yang Song James Ruse Agricultural High School NSW 11
Kevin Xian James Ruse Agricultural High School NSW 10
Jeremy Yip Trinity Grammar School VIC 11
Henry Yoo Perth Modern School WA 11
Juniors
Adam Bardrick Whitefriars College VIC 8
Rachel Hauenschild Kenmore State High School QLD 9
William Hu Christ Church Grammar School WA 8
Shivasankaran Jayabalan Rossmoyne Senior High School WA 8
Tony Jiang Scotch College VIC 9
Sharvil Kesarwani Merewether High School NSW 7
Ilia Kucherov Westall Secondary College VIC 10
Adrian Law James Ruse Agricultural High School NSW 9
Charles Li Camberwell Grammar School VIC 8
Jack Liu Brighton Grammar School VIC 8
Isabel Longbottom Rossmoyne Senior High School WA 9
Hilton Nguyen Sydney Technical High School NSW 8
Madeline Nurcombe Cannon Hill Anglican College QLD 10
Zoe Schwerkolt Fintona Girls' School VIC 10
Katrina Shen James Ruse Agricultural High School NSW 7
Eric Sheng Newington College NSW 10
Wen Zhang St Joseph's College QLD 8

* Equivalent to year 10 in Australia.

25
THE 2015 AUSTRALIAN MATHEMATICAL
OLYMPIAD
AUSTRALIAN MATHEMATICAL OLYMPIAD
DAY 1
Tuesday, 10 February 2015
Time allowed: 4 hours
No calculators are to be used.
Each question is worth seven points.

1. Define the sequence a1 , a2 , a3 , . . . by a1 = 4, a2 = 7, and

an+1 = 2an an1 + 2, for n 2.

Prove that, for every positive integer m, the number am am+1 is a term of the sequence.

2. For each positive integer n, let s(n) be the sum of its digits. We call a number nifty if it
can be expressed as n s(n) for some positive integer n.
How many positive integers less than 10,000 are nifty?

3. Let S be the set of all two-digit numbers that do not contain the digit 0. Two numbers
in S are called friends if their largest digits are equal and the difference between their
smallest digits is 1. For example, the numbers 68 and 85 are friends, the numbers 78
and 88 are friends, but the numbers 58 and 75 are not friends.
Determine the size of the largest possible subset of S that contains no two numbers that
are friends.

4. Let be a fixed circle with centre O and radius r. Let B and C be distinct fixed points
on . Let A be a variable point on , distinct from B and C. Let P be the point such
that the midpoint of OP is A. The line through O parallel to AB intersects the line
through P parallel to AC at the point D.

(a) Prove that, as A varies over the points of the circle (other than B or C), D lies
on a fixed circle whose radius is greater than or equal to r.
(b) Prove that equality occurs in part (a) if and only if BC is a diameter of .

c 2015 Australian Mathematics Trust




26
OLYMPIAD

DAY 2
Wednesday, 11 February 2015
Time allowed: 4 hours
No calculators are to be used.
Each question is worth seven points.

5. Let ABC be a triangle with ACB = 90 . The points D and Z lie on the side AB such
that CD is perpendicular to AB and AC = AZ. The line that bisects BAC meets CB
and CZ at X and Y , respectively.
Prove that the quadrilateral BXY D is cyclic.

6. Determine the number of distinct real solutions of the equation

(x 1) (x 3) (x 5) (x 2015) = (x 2) (x 4) (x 6) (x 2014).

7. For each integer n 2, let p(n) be the largest prime divisor of n.


Prove that there exist infinitely many positive integers n such that
  
p(n + 1) p(n) p(n) p(n 1) > 0.

8. Let n be a given integer greater than or equal to 3. Maryam draws n lines in the plane
such that no two are parallel.
For each equilateral triangle formed by three of the lines, Maryam receives three apples.
For each non-equilateral isosceles triangle formed by three of the lines, she receives one
apple.
What is the maximum number of apples that Maryam can obtain?

c 2015 Australian Mathematics Trust




27
AUSTRALIAN MATHEMATICAL OLYMPIAD SOLUTIONS

1. For reference, the sequence a1 , a2 , a3 , . . . is given by a1 = 4, a2 = 7, and

an+1 = 2an an1 + 2, for n 2.

Solution 1 (Linus Cooper, year 9, James Ruse Agricultural High School, NSW)
First we prove the following formula by induction.

an = n2 + 3, for n 1.

We require two base cases to get started. The formula is true for n = 1 and n = 2
because a1 = 4 = 12 + 3 and a2 = 7 = 22 + 3.
For the inductive step, assume that the formula is true for n = k 1 and n = k.
Then for n = k + 1, we have

ak+1 = 2ak ak1 + 2 (given)


2 2
= 2(k + 3) ((k 1) + 3) + 2 (inductive assumption)
= k 2 + 2k + 4
= (k + 1)2 + 3.

Hence the formula is also true for n = k + 1. This completes the induction.
Using the formula, we calculate

am am+1 = (m2 + 3)((m + 1)2 + 3)


= (m2 + 3)(m2 + 2m + 4)
= m4 + 2m3 + 7m2 + 6m + 12
= (m2 + m + 3)2 + 3
= am2 +m+3 .

Hence am am+1 is a term of the sequence. 

17

28
Solution 2 (Jack Liu, year 9, Brighton Grammar School, VIC)
The given rule for determining an+1 from an and an1 can be rewritten as

an+1 an = an an1 + 2, for n 2.

Thus the difference between consecutive terms of the sequence increases by 2 as we


go up. Hence starting from a2 a1 = 3, we may write down the following.

a2 a1 = 3
a3 a2 = 5
a4 a3 = 7
..
.
am am1 = 2m 1

If we add all these equations together, we find that almost everything cancels on
the LHS, and we have

am a1 = 3 + 5 + + 2m 1.

Using the standard formula11 for summing an arithmetic progression, we find

(2m + 2)(m 1)
am a1 =
2
2
= m 1.

Since a1 = 4, it follows that am = m2 + 3 for each m 1.


It is now a simple matter to calculate

am am+1 = (m2 + 3)((m + 1)2 + 3)


= m4 + 2m3 + 7m2 + 6m + 12
= (m2 + m + 3)2 + 3.

Hence am am+1 = an , where n = m2 + m + 3. Therefore, am am+1 is a term of the


sequence. 

1
1
The standard formula for the sum a + (a + d) + (a + 2d) + + (a + kd) is (2a+kd)(k+1)
. In our case
2
a = 3, d = 2 and k = m 2.

18

29
Solution 3 (Michael Robertson, year 11, Dickson College, ACT)
Starting from am and am+1 , let us compute the next few terms of the sequence in
terms of am and am+1 .

am+2 = 2am+1 am + 2
= 2(am+1 + 1) am

am+3 = 2am+2 am+1 + 2


= 2((2am+1 + 1) am ) am+1 + 2
= 3(am+1 + 2) 2am

Similarly, am+4 = 4(am+1 + 3) 3am .


We prove the following general formula by induction.

am+k = k(am+1 + k 1) (k 1)am , for k = 0, 1, 2, . . .. (1)

By inspection, formula (1) is true for the base cases k = 0 and k = 1.


For the inductive step, assume that formula (1) is true for k = r 1 and k = r.
Then for k = r + 1, we have

am+r+1 = 2am+r am+r1 + 2


= 2(r(am+1 + r 1) (r 1)am )
((r 1)(am+1 + r 2) (r 2)am ) + 2
= (r + 1)(am+1 + r) ram .

Hence the formula is also true for k = r + 1. This completes the induction.
Let us substitute k = am into formula (1). We have

am+am = am (am+1 + am 1) (am 1)am


= am am+1 .

Hence am am+1 = ak , where k = m + am . Therefore, am am+1 is a term of the


sequence. 

Comment This proof shows that the conclusion of the problem remains true for
any starting values a1 and a2 of the sequence, provided that m + am 0 for all m.
This is certainly true whenever a2 a1 0.

19

30
2. Solution 1 (Yang Song, year 12, James Ruse Agricultural High School, NSW)
Answer: 1000
For each positive integer n, let f (n) = n s(n). We seek the number of different
values that f (n) takes in the range from 1 up to 9999.
Lemma The function f has the following two properties.

(i) f (n + 1) = f (n) if the last digit of n is not a 9.


(ii) f (n + 1) > f (n) if the last digit of n is a 9.

Proof If the last digit of n is not a 9, then s(n + 1) = s(n) + 1. From this it easily
follows that f (n + 1) = f (n).
If the last digit of n is a 9, then suppose that the first k (k 1) digits from the
right-hand end of n are 9s, but the (k + 1)th digit from the right-hand end of n
is not a 9. In going from n to n + 1, the rightmost k digits all change from 9
to 0, and the (k + 1)th digit from the right-hand end of n increases by 1. Hence
s(n + 1) = s(n) 9k + 1, and so

f (n + 1) = n + 1 s(n + 1)
= n + 1 (s(n) 9k + 1)
= n s(n) + 9k
= f (n) + 9k
> f (n). 

From the lemma it follows that

f (1) = f (2) = = f (9) < f (10) = f (11) = = f (19)


< f (20) = f (21) = = f (29)
..
.
< f (10000) = f (10001) = = f (10009)
< f (10010).

Since, f (9) = 0, f (10) = 9, f (10000) = 9999 and f (10010) = 10008, the required
number of different values of f in the range from 1 up to 9999 is simply equal
to the number of multiples of 10 from 10 up to 10000. The number of these is
10000 10 = 1000. 

20

31
Solution 2 (Seyoon Ragavan, year 11, Knox Grammar School, NSW)
Suppose that the decimal representation of n is ak ak1 . . . a1 a0 where ak = 0. Then
we have the following.

n = 10k ak + 10k1 ak1 + + 101 a1 + a0


s(n) = ak + ak1 + + a1 + a0
n s(n) = (10k 1)ak + (10k1 1)ak1 + + (101 1)a1

If k 4, then n s(n) (10k 1)ak 9999. Note that 9999 is nifty, because if
n = 10000 then n s(n) = 9999. So it only remains to deal with positive integers
that are less than 9999.
If k 3, then n s(n) = 999a3 + 99a2 + 9a1 . It follows that the nifty numbers
less than 999 are precisely those numbers of the form 999a3 + 99a2 + 9a1 , where
a1 , a2 , a3 {0, 1, 2, . . . , 9}.
Lemma If 999a + 99b + 9c = 999d + 99e + 9f where a, b, c, d, e, f {0, 1, 2, . . . , 9},
then a = d, b = e and c = f .
Proof If 999a + 99b + 9c = 999d + 99e + 9f , then this can be rearranged as

111(a d) + 11(b e) + (c f ) = 0. (1)

If a > d, then since b e 9 and c f 9, we have LHS(1) 111 99 9 > 0.


A similar contradiction is reached if a < d. Hence a = d and equation (1) becomes

11(b e) + (c f ) = 0. (2)

If b > e, then since cf 9, we have LHS(2) 119 > 0. A similar contradiction


is reached if b < e. Hence b = e, from which c = f immediately follows. 
Returning to the problem, the number of combinations of a1 , a2 , a3 is 103 = 1000.
The lemma guarantees that each of these combinations leads to a different nifty
number. However, we exclude the combination a1 = a2 = a3 = 0 because although
0 is nifty, it is not positive.
In summary, there are 999 nifty numbers from the case k 3, and one nifty number
from the case k 4. This yields a total of 1000 nifty numbers. 

21

32
3. Solution 1 (Alexander Gunning, year 12, Glen Waverley Secondary College, VIC)
Answer: 45
Call a subset T of S friendless if no two numbers in T are friends. We visualise a
friendless subset T as follows. Draw a 9 9 square grid. If the number 10a + b is in
T we shade in the square that lies in the ath row and bth column.
First we exhibit a friendless subset T of size 45.
Let T consist of all two-digit numbers whose smaller digit is odd, as depicted in the
first diagram below. No two numbers in T are friends because their smaller digits
are both odd and hence cannot differ by 1. Since there are 45 shaded squares, we
have shown that |T | = 45 is possible.

1 2 3 4 5 6 7 8 9 1 2 3 4 5 6 7 8 9
1 1
2 2
3 3
4 4
5 5
6 6
7 7
8 8
9 9

Finally, we show that all friendless subsets T of S have at most 45 numbers.


To see this we pair up some of the squares in the 9 9 square grid as shown in the
second diagram above. Observe that the numbers in each pair are friends. Hence T
can have at most one number from each pair. Since there are 9 unpaired numbers
in the top row and 36 pairs, this shows that T has at most 45 numbers. 

22

33
Solution 2 (Anand Bharadwaj, year 9, Trinity Grammar School, VIC)
Consider the following nine lines of numbers.

11
21, 22, 12
31, 32, 33, 23, 13
41, 42, 43, 44, 34, 24, 14
51, 52, 53, 54, 55, 45, 35, 25, 15
61, 62, 63, 64, 65, 66, 56, 46, 36, 26, 16
71, 72, 73, 74, 75, 76, 77, 67, 57, 47, 37, 27, 17
81, 82, 83, 84, 85, 86, 87, 88, 78, 68, 58, 48, 38, 28, 28
91, 92, 93, 94, 95, 96, 97, 98, 99, 89, 79, 69, 59, 49, 39, 29, 19

Each pair of neighbouring numbers on any given line are friends. So a subset T of S
that contains no friends cannot include consecutive numbers on any of these lines.
On the ith line there are exactly 2i 1 integers. The maximum number of integers
we can choose from the ith line without choosing neighbours is i. This shows that
T contains at most 1 + 2 + 3 + 4 + 5 + 6 + 7 + 8 + 9 = 45 integers.
Furthermore, |T | = 45 only if we choose exactly i numbers from the ith line without
choosing neighbours. There is only one way to do this, namely take every second
number starting from the left of each line.
It is still necessary to verify that T contains no friends because even some non-
adjacent numbers from the same line, such as 31 and 23, can be friends. To do this,
note that the smaller digit of each number in T is odd. Thus for any pair of integers
in T , the difference between their smaller digits is even, and thus cannot be equal
to 1. Hence T contains no friends. 

Comment This proof shows that there is exactly one subset of S of maximal size
that contains no friends.

23

34
4. Comment All solutions that were dependent on how the diagram was drawn re-
ceived a penalty deduction of 1 point. The easiest way to avoid diagram dependence
was to use directed angles as in the three solutions we present here.
For any two lines m and n, the directed angle between them is denoted by (m, n).
This is the angle by which one may rotate m anticlockwise to obtain a line parallel
to n.22

Solution 1 (Yang Song, year 12, James Ruse Agricultural High School, NSW)
Let Q be the intersection of lines OC and P D. Since AC  P Q and A is the
midpoint of OP , it follows that C is the midpoint of OQ.

D
A

O
B C
Q

(a) We know OD  BA and DQ  AC. It follows that (OD, DQ) = (BA, AC),
which is fixed because A lies on . This implies D lies on a fixed circle, say,
through O and Q.
Let s be the radius of . Since the diameter is the largest chord length in a
circle, we have 2s OQ. Since OQ = 2OC = 2r we have s r, as desired. 
(b) From the preceding analysis, we have s = r if and only if OQ is a diameter of
. This is achieved if and only if OD DQ, which is equivalent to BA AC.
But the chords BA and AC of are perpendicular if and only if BC is a
diameter of . 

22 For
more details on how to work with directed angles, see the section Directed angles in chapter 17 of
Problem Solving Tactics published by the AMT.

24

35
Solution 2 (Ilia Kucherov, year 11, Westall Secondary College, VIC)
Let E be the intersection of lines AC and OD. Since AC  P D and A is the
midpoint of OP , it follows that E is the midpoint of OD.

A D

B C
Q

(a) As in solution 1, we use directed angles.


Since OE  BA, we have (OE, EC) = (BA, AC), which is fixed because
A lies on . This implies that E lies on a fixed circle, say, through O and
C. Since E is the midpoint of OD, the point D is the image of E under the
dilation centred at O and of enlargement factor 2. The image of under the
dilation is a circle that is twice as large as . Thus D lies on .
Let s be the radius of . Then has radius 2s . But OC is a chord of ,
hence its length r is less than or equal to the diameter s of . Hence s r, as
required. 
(b) From the preceding analysis we have s = r if and only if OC is a diameter of
. This is achieved if and only if OE EC, which is equivalent to BA AC.
But the chords BA and AC of are perpendicular if and only if BC is a
diameter of . 

25

36
Solution 3 (Kevin Xian, year 11, James Ruse Agricultural High School, NSW)
The motivation for this solution comes from considering a couple of special positions
for A.
If A is diametrically opposite C, then D = O. If A is diametrically opposite B, then
this gives a second position for D. With this in mind, let X be the point on that
is diametrically opposite B, and let Y be the point such that X is the midpoint of
OY .

D
A

Y
X
O

B C

(a) Using directed angles, we have

(OY, OD) = (BO, BA) (OD  BA)


= (AB, AO) (OB = OA)
= (OD, OP ). (BA  OD)

We also have OY = 2OX = 2OA = OP . Therefore, Y and P are symmetric


in the line OD. It follows that

(Y D, OD) = (OD, P D)
= (OD, OP ) + (OP, P D)
= (BA, OA) + (OA, AC)
(OD  BA and P D  AC)
= (BA, AC).

Hence (Y D, OD) is fixed because B, C and are fixed.


Since O and Y are also fixed points, it follows that D lies on a fixed circle,
say, that passes through O and Y .
Let s be the radius of . Then by the extended sine rule applied to ODY

26

37
we have
OY
2s =
sin (Y D, OD)
2r
=
sin (BA, AC)
2r.

Therefore s r, as required. 
(b) From the preceding analysis, we have s = r if and only if

sin(BA, AC) = 1.

This is equivalent to BA AC, which occurs if and only if BC is the diameter


of . 

27

38
5. This was the easiest problem of the competition. Of the 106 students who sat the
AMO, 77 found a complete solution. There were many different routes to a solution,
and we present some of them here.
Solution 1 (Zoe Schwerkolt, year 11, Fintona Girls School, VIC)
We are given ACZ is isosceles with AC = AZ. By symmetry, the angle bisector
at A is also the altitude from A, and so AX CZ. Thus CY A = 90 = CDA.
It follows that ADY C is a cyclic quadrilateral.

X
Y

x
x

A D Z B

Since ADY C is cyclic, we may set Y DC = Y AC = x, as in the diagram. Hence

BDY = 90 x. (1)

But from the exterior angle sum in CAX, we have AXB = 90 + x, and so

Y XB = 90 + x. (2)

Adding (1) and (2) yields

BDY + Y XB = 180 ,

and so BXY D is a cyclic quadrilateral. 

28

39
Solution 2 (Alan Guo, year 12, Penleigh and Essendon Grammar School, VIC)
We deduce that ADY C is cyclic as in solution 1.
It follows that

AY D = ACD (ADY C cyclic)


= 90 DAC (angle sum CAD)
= 90 BAC
= CBA. (angle sum ABC)

Hence AY D = XBD, and so BXY D is cyclic.

X
Y

A D Z B

29

40
Solution 3 (Wen Zhang, year 9, St Josephs College, QLD)

We are given ACZ is isosceles with AC = AZ. Hence by symmetry, the angle
bisector at A is both the median and the altitude from A. Thus Y is the midpoint
of CZ.
Since CDZ is right-angled at D, the point Y is the circumcentre of CDZ. Hence
we have Y C = Y D = Y Z.
Let CBA = XBD = 2x. We now compute some other angles in the diagram.

BAC = 90 2x (angle sum ABC)


ZAY = 45 x (AY bisects BAC)
Y ZA = 45 + x (angle sum AZY )
ZDY = 45 + x (Y D = Y Z)
DY Z = 90 2x (angle sum Y DZ)
DY X = 180 2x (AX CZ)

Hence XBD + DY X = 180 , and so BXY D is cyclic.

X
Y

2x
A D Z B

30

41
Solution 4 (Seyoon Ragavan, year 11, Knox Grammar School, NSW)
As in solution 1, we deduce that AY CZ.
Let lines AY and CD intersect at H. Since AY CZ and CD AZ, it follows that
H is the orthocentre of CAZ. Hence ZH AC. But since AC BC, we have
ZH  BC. Observe also that HDZY is cyclic because ZDH = 90 = HY Z.
We now have

CXH = ZHX (BC  ZH)


= ZHY
= ZDY. (HDZY cyclic)

Hence CXY = BDY , and so BXY D is cyclic.

X
Y
H

A D Z B

31

42
Solution 5 (Kevin Xian, year 11, James Ruse Agricultural High School, NSW)
As in solution 1, we deduce that AY CZ. Hence using the angle sums in CAY
and CAX, we have

ACY = 90 XAC = CXY.

Hence by the alternate segment theorem, line AC is tangent to circle CY X at C.


Using the power of point A with respect to circle CY X, we have

AC 2 = AY AX. (1)

In a similar way we may use the angle sums in CAD and ABC to find

ACD = 90 BAC = CBD.

Hence by the alternate segment theorem, line AC is tangent to circle CDB at C.


Using the power of point A with respect to circle CDB, we have

AC 2 = AD AB. (2)

Combining (1) and (2), it follows that

AY AX = AD AB,

and so BXY D is cyclic.

X
Y

A D Z B

32

43
6. Solution 1 (Michelle Chen, year 11, Methodist Ladies College, VIC)
Answer: 1008
Let f (x) = p(x) q(x), where

p(x) = (x 1)(x 3)(x 5) (x 2015),


q(x) = (x 2)(x 4)(x 6) (x 2014).

We seek the number of distinct real solutions to the equation f (x) = 0. Note that
f (x) is a polynomial of degree 1008 because p(x) has degree 1008 while q(x) has
degree 1007.
Observe that p(x) is the product of an even number of brackets and q(x) is the
product of an odd number of brackets. Therefore,

f (0) = (1) (3) (2015)


(2) (4) (2014)
= 1 3 2015 + 2 4 2014
> 0.
We also have
f (2016) = 2015 2013 1 2014 2012 2
> 0,

where the last line is true because 2015 > 2014, 2013 > 2012, and so on down to
3 > 2.
Next, for x = 2, 4, 6, . . . , 2014, we have q(x) = 0, and so f (x) = p(x). Hence

f (2) = 1 (1) (3) (2013) < 0.

Note that by increasing the value of x from x = 2 to x = 4, we reduce the number


of negative brackets in the product for q(x) by 1. This has the effect of changing
the sign of f (x). In general, each time we increase x by 2 up to a maximum of
x = 2014, we change the sign of f (x). Therefore, we have the following inequalities.

f (0) > 0
f (2) < 0
f (4) > 0
f (6) < 0
..
.
f (2014) < 0
f (2016) > 0

Since f (x) is a polynomial it is a continuous function. Hence, by the interme-


diate value theorem, f (x) = 0 has at least one solution in each of the intervals
(0, 2), (2, 4), (4, 6), . . . , (2014, 2016). Therefore, there are at least 1008 different so-
lutions to f (x) = 0.
Finally, since f (x) is a polynomial of degree 1008, the equation f (x) = 0 has at
most 1008 roots. Hence we may conclude that f (x) = 0 has exactly 1008 distinct
real solutions. 

33

44
Solution 2 (Yong See Foo, year 11, Nossal High School, VIC)
The polynomials p(x), q(x) and f (x) are defined as in solution 1. We also deduce
f (2016) > 0, as in solution 1.
For any x {1, 3, 5, . . . , 2015}, we have p(x) = 0, and so f (x) = q(x). Therefore,
for any x {1, 3, 5, . . . , 2015}, we have
f (x) = (x 2)(x 4) (x (x 1)) (x (x + 1)) (x 2014).
(We use the symbol merely as a placeholder for future reference.)
All bracketed terms to the left of are positive while the remaining bracketed terms
to the right of are negative. Hence the total number of bracketed terms that are
negative is equal to 2014(x+1)
2
+ 1 = 2015x
2
.33 Taking into account the minus sign at
the front, it follows that for each x {1, 3, 5, . . . , 2015}, we have
f (x) > 0 for x 1 (mod 4) and f (x) < 0 for x 3 (mod 4).
Hence we have the following inequalities.
f (1) > 0
f (3) < 0
f (5) > 0
..
.
f (2013) > 0
f (2015) < 0
f (2016) > 0

Since f (x) is a polynomial, it is a continuous function. Hence by the interme-


diate value theorem, f (x) = 0 has at least one solution in each of the intervals
(1, 3), (3, 5), . . . , (2013, 2015), (2015, 2016). Therefore, there are at least 1008 differ-
ent solutions to the equation f (x) = 0.
However, as in solution 1, f (x) is a polynomial of degree 1008, and so f (x) = 0 has
at most 1008 roots. Thus we may conclude that f (x) = 0 has exactly 1008 distinct
real solutions. 

Comment Some contestants found it helpful to do a rough sketch of p(x) and q(x).
This helps to determine the signs of p(x), q(x) and f (x) for x = 0, 1, 2, . . . , 2016.
From this one can further narrow down the location of the solutions to f (x) = 0.
They are found in the intervals (1, 2), (3, 4), (5, 6), . . . , (2013, 2014), (2015, 2016).
y y = p(x)
y = q(x)

... x
1 2 3 4 5 6 7 2013 2014 2015 2016

3
3
This is all still true in the extreme cases x = 1 and x = 2015. The case x = 1 corresponds to putting the
to the left of the first term (x 2) but to the right of the negative sign. The case x = 2015 corresponds
to putting the to the right of the last term (x 2014).

34

45
7. There are many different ways to solve this problem. Over 10 different methods of
solution were found among the 24 contestants who completely solved the problem.
Some of these are presented here.

Solution 1 (Charles Li, year 9, Camberwell Grammar School, VIC)


We proceed by contradiction. Assume that only finitely many integers n satisfy
  
p(n + 1) p(n) p(n) p(n 1) > 0.

Observe that p(n) = p(n + 1) for all positive integers n 2 because adjacent
numbers cannot be divisible by the same prime. Hence there is a positive integer N
such that
  
p(n + 1) p(n) p(n) p(n 1) < 0 for all integers n > N . (*)

Thus p(n) > p(n 1) if and only if p(n + 1) < p(n) for all n > N . It follows that
the graph of p(n) versus n alternates between peaks and valleys once n > N . This
can be schematically visualised as follows.

p(n)

Consider the number 2m where m > 1 is large enough so that 2m > N . Note that
p(2m ) = 2. Since 2m 1 and 2m +1 are odd we have p(2m +1) > 2 and p(2m 1) > 2.
Hence the above graph has a valley at 2m . Since peaks and valleys alternate for
every n > N , it follows that there is a valley at every even number and a peak at
every odd number once we pass N .
Consider the number 3m . Note that p(3m ) = 3. However, from the preceding
paragraph, there is a peak at 3m because it is odd and greater than N . Hence
p(3m 1) = 2 and p(3m + 1) = 2. This is possible if and only if 3m 1 and 3m + 1
are powers of 2. But the only powers of 2 that differ by 2 are 21 and 22 . This implies
m = 1, which is a contradiction. 

Comment 1 Some contestants found a second way to deduce that the graph of
p(n) versus n (for n > N ) has peaks at odd n and valleys at even n.
They observed that if q > 3 is a prime number greater than N , then there is a peak
at q. This is because p(q) = q, while p(q + 1) q+1
2
< q and p(q 1) q1
2
< q.
Comment 2 All solutions used the method of indirect proof.44 They all established
that the graph of p(n) versus n (for n > N ) has peaks at odd values of n and valleys

4
4
Also known as proof by contradiction.

35

46
at even values of n. We present some of the variations that contestants used to
derive a contradiction from this point on.

Variation 1 (Isabel Longbottom, year 10, Rossmoyne Senior High School, WA)
Consider the number 32m+1 , where m 1 satisfies 32m+1 > N .
Since 32m+1 is odd and p(32m+1 ) = 3, it follows that 32m+1 1 is a power of 2.
Since m > 1, the number 32m+1 1 is a power of 2 which is greater than 2. Hence
32m+1 1 (mod 4).
But 32m+1 (1)2m+1 1  1 (mod 4), which is a contradiction. 

Variation 2 (Richard Gong, year 10, Sydney Grammar School, NSW)


Consider the number 34m , where m 1 is large enough so that 34m > N .
Since 34m is odd and p(34m ) = 3, it follows that 34m 1 is a power of 2.
But 34m = 81m 1m (mod 5).
So 5 | 34m 1, which is a contradiction. 

Variation 3 (Anthony Pisani, year 8, St Pauls Anglican Grammar School, VIC)


By Dirichlets theorem there are infinitely many primes of the form q = 6k + 1.
Choose any such prime q > N .
There is a valley at 2q because it is even. Hence p(2q + 1) > p(2q) = q.
But 2q + 1 = 2(6k + 1) + 1 = 3(4k + 1), and so 2q + 1 is a multiple of 3.
Hence p(2q + 1) 2q+1
3
< q, which is a contradiction. 

Variation 4 (Jack Liu, year 9, Brighton Grammar School, VIC)


Consider the number 2q, where q > 3 is prime greater than N . Note that p(2q) = q.
There is a valley at 2q because it is even.
However, 2q 1, 2q and 2q + 1 are three consecutive numbers, and so one of them
is a multiple of 3. Since q > 3 we know 3  2q.
2q1 2q+1
If 3 | 2q 1, then p(2q 1) 3
< q, and if 3 | 2q + 1, then p(2q + 1) 3
< q.
Either way, this contradicts that there is a valley at 2q. 

Variation 5 (Kevin Shen, year 1255 , Saint Kentigern College, NZ)


Let q > N be any prime. There is a valley at 2q because it is even.
So we have p(2q 1) > p(2q) = q. This implies that p(2q 1) = 2q 1.
Thus 2q1 is prime whenever q is prime. Therefore, we also have 2(2q1)1 = 4q3
is prime.
A simple induction shows that 2r (q 1) + 1 is prime for r = 0, 1, 2, . . ..
In particular 2q1 (q 1) + 1 is prime.
By Fermats little theorem, 2q1 (q 1) + 1 1(q 1) + 1 0 (mod q).
Hence q | 2q1 (q 1) + 1. Since 2q1 (q 1) + 1 > q, we have contradicted that
2q1 (q 1) + 1 is prime. 

5
5
Equivalent to year 11 in Australia.

36

47
Variation 6 (Thomas Baker, year 11, Scotch College, VIC)
Let q1 < q2 < be the list of all odd primes in order starting from q1 = 3.
By Dirichlets theorem there are infinitely many primes of the form 4k + 3. So we
may choose j > 3 such that qj > N and such that the list q1 , q2 , . . . , qj contains an
odd number of primes of the form of 4k + 3.
Consider the number m = q1 q2 qj . Note that p(m) = qj . Furthermore, there is a
peak at m because it is odd.
Note m 3 (mod 4). Thus m 1 = 2r for some odd number r.
Furthermore, r is not divisible by any qi (1 i j). Hence p(m 1) is a prime
that is greater than pj . Thus p(m 1) > p(m), which contradicts that there is a
peak at m. 

37

48
Solution 2 (Jeremy Yip, year 12, Trinity Grammar School, VIC)
We shall prove that p(n 1) < p(n) < p(n + 1) for infinitely many n. In particular,
for each odd prime q, we show there exists a positive integer k such that
k k k
p(q 2 1) < p(q 2 ) < p(q 2 + 1). (1)

a b
Lemma 1 For any odd prime q, we have gcd(q 2 + 1, q 2 + 1) = 2 for all integers
1 a < b.
a b
Proof Let d = gcd(q 2 + 1, q 2 + 1). Using the difference of perfect squares factori-
sation we know x 1 | x2 1 and x + 1 | x2 1. From this we deduce the following
chain of divisibility.
a a+1
q2 + 1 | q2 1
2a+1 2a+2
q 1|q 1
2a+2 2a+3
q 1|q 1
..
.
b1 b
q2 1 | q2 1
a b b b
It follows that q 2 + 1 | q 2 1, and so d | q 2 1. Combining this with d | q 2 + 1

a b
yields d | 2. Since q 2 + 1 and q 2 + 1 are even, we conclude that d = 2.
Lemma 2 If (1) is false for all positive integers k, then
k k
p(q 2 1) < q and p(q 2 + 1) < q for k = 1, 2, . . . .

Proof We proceed by induction under the assumption that (1) is false.


 
For the base case k = 1, since q is odd we have q 2 1 = 2(q 1) q+1 2
. Hence
2 2
p(q 1) < q. However, since (1) is false for k = 2, we also have p(q + 1) < q.
For the inductive step, suppose we know that
j j
p(q 2 1) < q and p(q 2 + 1) < q,
j+1 j j
for some positive integer j. Then since q 2 1 = (q 2 1)(q 2 + 1), we have
j+1 j+1
p(q 2 1) < q. However, since (1) is false for k = j+1, we also have p(q 2 +1) < q.
This completes the induction. 
To complete the proof of the problem, let us focus on the following list of numbers.

L = p(q 2 + 1), p(q 4 + 1), p(q 8 + 1), . . .

From lemma 1, q 2 + 1, q 4 + 1, q 8 + 1, . . . all have pairwise greatest common divisor


equal to 2. Hence at most one of the numbers in L is equal to 2, while the others
are distinct odd primes. This means that L contains infinitely many different prime
numbers. This is a contradiction because lemma 2 implies that L contains only
finitely many different numbers. 

38

49
Comment Jeremys proof is rather impressive because it establishes a much
stronger result than what was required. The AMO problem only required a proof
that at least one of p(n 1) < p(n) < p(n + 1) and p(n 1) > p(n) > p(n + 1)
occurs infinitely often, but without pinning down which of the two alternatives does
occur infinitely often. Jeremys proof establishes that p(n 1) < p(n) < p(n + 1)
definitely occurs infinitely often.66

66 Erdos
and Pomerance proved this result in their 1978 research paper On the largest prime factors of n
and n + 1. The question of whether p(n 1) > p(n) > p(n + 1) could occur infinitely often was finally
resolved in the positive by Balog in his 2001 research paper On triplets with descending largest prime
factors.

39

50
8. This was the most difficult problem of the 2015 AMO. Just four contestants managed
to solve it completely.

Solution (Seyoon Ragavan, year 11, Knox Grammar School, NSW)


 
Answer: n n1
2

We shall refer to a triangle ABC as being isosceles with base BC if AB = AC.


Note that if ABC is equilateral then it is isosceles with base AB, isosceles with
base BC, and isosceles with base AC. In this way an equilateral triangle counts as
three isosceles triangles depending on which side is considered the base. Thus the
number of apples Maryam receives is equal to the number of base-specified isosceles
triangles.
Let S denote the set of lines that Maryam draws. Consider a line a S. We
determine how many ways a can be the base of an isosceles triangle. Observe that
if a, b, c and a, d, e both form isosceles triangles with base a, then since no two lines
are parallel,
  the pairs {b, c} and {d, e} are either equal or disjoint. Thus there are at
most n1 2
pairs {b, c} from S such that a, b, c forms an isosceles triangle with base
a. Since there are n possibilities fora, we conclude that the number of base-specified
isosceles triangle is at most n n1 2
.
It remains to show that this is attainable. Take n equally spaced lines passing

through the origin so that the angle between consecutive lines is 180
n
. Next translate
each of the lines so that no
 n1  three of them are concurrent. We claim this configuration
of lines results in n 2 base-specified isosceles triangles.
For any line x, let x denote the line after it has been translated. Consider any
triangle bounded by lines a , b , c . It is isosceles with base a if and only if before the
translations, the lines b and c were symmetric in a. There are n ways of choosing
 n1
a. Once a is chosen, then by our construction there are 2 pairs b, c such that
they are symmetric in a. Indeed, if n is odd, all the other lines are involved in
a symmetric pair about a. If n is even then exactly one line is not involved in a
symmetric pair about a because it is perpendicular to a. Thus  the total number of
base specified isosceles triangles for this construction is n n1 2
, as desired. 

The following diagram illustrates the construction for n = 6. Each line is a base for
two isosceles triangles, one of which is equilateral. In total there are two equilateral
triangles and six isosceles triangles, making twelve base-specified isosceles triangles
in all.

40

51
 
Comment Other constructions that achieve the bound n n1 2
are possible. The
following one is by Andrew Elvey Price, Deputy Leader of the 2015 Australian IMO
team.
If n is odd, Maryam can achieve this by drawing the lines to form the sides of a
regular polygon with n sides. If n is even, Maryam can achieve this by drawing the
lines to form all but one of the sides of a regular polygon with n + 1 sides. 

41

52
AUSTRALIAN MATHEMATICAL OLYMPIAD STATISTICS
Score Distribution/Problem

Number of Problem Number


Students/Score 1 2 3 4 5 6 7 8

0 10 14 29 80 17 60 47 86

1 7 2 1 8 0 1 30 8

2 1 7 18 0 6 3 1 1

3 2 10 4 0 5 4 4 4

4 13 10 11 1 0 5 0 0

5 6 9 9 0 0 1 0 3

6 17 26 11 8 1 7 0 0

7 50 28 23 9 77 25 24 4

Average Mark 5.2 4.6 3.4 1.2 5.4 2.5 2.0 0.6

53
AUSTRALIAN MATHEMATICAL OLYMPIAD RESULTS

Name School Year

Perfect Score and Gold

Alexander Gunning Glen Waverley Secondary College VIC 12

Seyoon Ragavan Knox Grammar School NSW 11

Gold

Jeremy Yip Trinity Grammar School VIC 12

Yong See Foo Nossal High School VIC 11

Ilia Kucherov Westall Secondary College VIC 11

Yang Song James Ruse Agricultural High School NSW 12

Allen Lu Sydney Grammar School NSW 12

Alan Guo Penleigh and Essendon Grammar School VIC 12

Thomas Baker Scotch College VIC 11

Richard Gong Sydney Grammar School NSW 10

Henry Yoo Perth Modern School WA 12

Silver

Matthew Cheah Penleigh and Essendon Grammar School VIC 10

Kevin Xian James Ruse Agricultural High School NSW 11

Kevin Shen Saint Kentigern College NZ 12 NZ

George Han Westlake Boys High School NZ 13 NZ

Wilson Zhao Killara High School NSW 11

Michelle Chen Methodist Ladies' College VIC 11

Leo Li Christ Church Grammar School WA 11

Jack Liu Brighton Grammar School VIC 9

Charles Li Camberwell Grammar School VIC 9

Anthony Pisani St Paul's Anglican Grammar School VIC 8

Ivan Zelich Anglican Church Grammar School QLD 12

Michael Robertson Dickson College ACT 11

Edward Chen Palmerston North Boys High School NZ 11 NZ

William Hu Christ Church Grammar School WA 9

Martin Luk King's College NZ 13 NZ

54
Name School Year

Bronze

Jerry Mao Caulfield Grammar School VIC 9

Zoe Schwerkolt Fintona Girls' School VIC 11

Harish Suresh James Ruse Agricultural High School NSW 11

Justin Wu James Ruse Agricultural High School NSW 11

Austin Zhang Sydney Grammar School NSW 10

Isabel Longbottom Rossmoyne Senior High School WA 10

Anand Bharadwaj Trinity Grammar School VIC 9

Devin He Christ Church Grammar School WA 11

Miles Yee-Cheng Lee Auckland International College NZ 12 NZ

Bobby Dey James Ruse Agricultural High School NSW 10

Tony Jiang Scotch College VIC 10

Nitin Niranjan All Saints Anglican School QLD 12

William Song Scotch College VIC 11

Wen Zhang St Joseph's College, Gregory Terrace QLD 9

Prince Michael Balanay Botany Downs Secondary College NZ 13 NZ

Michael Chen Scotch College VIC 12

William Wang King's College NZ 12 NZ

Xuzhi Zhang Auckland Grammar School NZ 13 NZ

Keiran Lewellen Home Schooled NZ 10 NZ

Eryuan Sheng Newington College NSW 11

Linus Cooper James Ruse Agricultural High School NSW 9

David Steketee Hale School WA 12

Alexander Barber Scotch College VIC 11

Matthew Jones All Saints Anglican School QLD 12

Madeline Nurcombe Cannon Hill Anglican College QLD 11

Steven Lim Hurlstone Agricultural High School NSW 9

55
27TH ASIAN PACIFIC MATHEMATICS OLYMPIAD

March, 2015
Time allowed: 4 hours
No calculators are to be used
Each problem is worth 7 points

Problem 1. Let ABC be a triangle, and let D be a point on side BC. A line
through D intersects side AB at X and ray AC at Y . The circumcircle of triangle
BXD intersects the circumcircle of triangle ABC again at point Z = B. The lines
ZD and ZY intersect again at V and W , respectively. Prove that AB = V W .
Problem 2. Let S = {2, 3, 4, . . .} denote the set of integers that are greater than or
equal to 2. Does there exist a function f : S S such that
f (a)f (b) = f (a2 b2 ) for all a, b S with a = b?

Problem 3. A sequence of real numbers a0 , a1 , . . . is said to be good if the following


three conditions hold.
(i) The value of a0 is a positive integer.
ai
(ii) For each non-negative integer i we have ai+1 = 2ai + 1 or ai+1 = .
ai + 2
(iii) There exists a positive integer k such that ak = 2014.
Find the smallest positive integer n such that there exists a good sequence a0 , a1 , . . .
of real numbers with the property that an = 2014.
Problem 4. Let n be a positive integer. Consider 2n distinct lines on the plane, no
two of which are parallel. Of the 2n lines, n are colored blue, the other n are colored
red. Let B be the set of all points on the plane that lie on at least one blue line,
and R the set of all points on the plane that lie on at least one red line. Prove that
there exists a circle that intersects B in exactly 2n 1 points, and also intersects R
in exactly 2n 1 points.
Problem 5. Determine all sequences a0 , a1 , a2 , . . . of positive integers with a0 2015
such that for all integers n 1:
(i) an+2 is divisible by an ;
(ii) |sn+1 (n + 1)an | = 1, where sn+1 = an+1 an + an1 + (1)n+1 a0 .

56
27TH ASIAN PACIFIC MATHEMATICS OLYMPIAD
SOLUTIONS
1. Solution (Alan Guo, year 12, Penleigh and Essendon Grammar School, VIC)
Applying the Pivot theorem to AXY we see that circles ABC, XBD and CDY
are concurrent at point Z.11 Hence CDZY is cyclic.

D C
B

W
Z Y

Let (m, n) denote the directed angle between any two lines m and n.22 We have

(W Z, V Z) = (Y Z, DZ)
= (Y C, DC) (CDZY cyclic)
= (AC, BC).

Therefore V W and AB subtend equal or supplementary angles in . It follows that


V W = AB. 

Comment All solutions that were dependent on how the diagram was drawn re-
ceived a penalty deduction of 1 point. The easiest way to avoid diagram dependence
was to use directed angles as in the solution presented above.

11
The point Z is also the Miquel point of the four lines AB, AY , BC and XY . It is the common point of
the circumcircles of ABC, AXY , BDX and CDY . See the sections entitled Pivot theorem and
Four lines and four circles found in chapter 5 of Problem Solving Tactics published by the AMT.
22
This is the angle by which one may rotate m anticlockwise to obtain a line parallel to n. For more
details, see the section Directed angles in chapter 17 of Problem Solving Tactics published by the AMT.

43

57
2. Solution 1 (Henry Yoo, year 12, Perth Modern School, WA)
Answer: No
Assume such a function exists. For any positive integer n we have
 
f (2n )f (2n+4 ) = f 22(2n+4) = f (2n+1 )f (2n+3 ),

and so
f (2n ) f (2n+3 )
= . (1)
f (2n+1 ) f (2n+4 )
Similarly,  
f (2n+1 )f (2n+4 ) = f 22(2n+5) = f (2n+2 )f (2n+3 ),
and so
f (2n+1 ) f (2n+3 )
= . (2)
f (2n+2 ) f (2n+4 )
Combining (1) and (2), we find that

f (2n ) f (2n+1 )
= . (3)
f (2n+1 ) f (2n+2 )

Since (3) is true for all positive integers n, it follows that

f (21 ), f (22 ), f (23 ), . . .

is a geometric sequence. Thus f (2n ) = arn1 for some positive rational numbers a
and r.
Since f (2)f (22 ) = f (26 ) we have

a ar = ar5
a = r4 . (4)

And since f (2)f (23 ) = f (28 ) we have

a ar2 = ar7
a = r5 . (5)

Comparing (4) and (5) we have r4 = r5 . Since r > 0, we have r = 1. But then from
(4) we have a = 1. Thus f (2) = 1  S, a contradiction. Hence there are no such
functions. 

44
58
Solution 2 (Linus Cooper, year 9, James Ruse Agricultural High School, NSW)
Suppose such a function exists. Then for any integer b > 2 we have

f (8b)f (b) = f (64b4 ) = f (2)f (4b2 ) = f (2)f (2)f (b),

and so
f (8b) = f (2)2 .
Thus f (c) = f (2)2 whenever c 24 and c is a multiple of 8.
Next we have

f (24)f (32) = f (242 322 )


f (2)4 = f (2)2 .

Since f (2) > 0, we have f (2) = 1  S. This contradiction shows that no such
function exists. 

45

59
Solution 3 (Jeremy Yip, year 12, Trinity Grammar School, VIC)
Suppose such a function exists. Let a < b < c be positive integers. We work with
the quantity f (2a )f (2b )f (2c ) in two different ways.
 a 
f (2 )f (2c ) f (2b ) = f (22a+2c )f (2b ) (since c > a)
= f (24a+2b+4c ) (since 2a + 2c > b) (1)
 
f (2b )f (2c ) f (2a ) = f (22b+2c )f (2a ) (since c > b)
= f (22a+4b+4c ) (since 2b + 2c > a) (2)
 
f (2a )f (2b ) f (2c ) = f (22a+2b )f (2c ) (since b > a)
= f (24a+4b+2c ) (if c = 2a + 2b) (3)

From (1), (2) and (3) it follows that

f (42a+2b+c ) = f (42a+b+2c ) = f (4a+2b+2c ), (4)

for any three positive integers a < b < c satisfying c = 2a + 2b.


In what follows we use (4) repeatedly.

(a, b, c) = (1, 2, 3) f (49 ) = f (410 ) = f (411 )


(a, b, c) = (1, 2, 4) f (410 ) = f (412 ) = f (413 )
(a, b, c) = (1, 3, 5) f (413 ) = f (415 ) = f (417 )
(a, b, c) = (1, 4, 7) f (417 ) = f (420 ) = f (423 )

Thus f (49 ) = f (420 ).


However, f (49 )f (4) = f (420 ), and so f (4) = 1  S. This contradiction shows that
no such function exists. 

46

60
Solution 4 (Angelo Di Pasquale, Director of Training, AMOC)
For any a, b S, choose an integer c > a, b. Then since bc > a and c > b, we have

f (a4 b4 c4 ) = f (a2 )f (b2 c2 ) = f (a2 )f (b)f (c).

Furthermore, since ac > b and c > a, we have

f (a4 b4 c4 ) = f (b2 )f (a2 c2 ) = f (b2 )f (a)f (c).

Comparing these two equations, we find that for all a, b S,

2 2 f (a2 ) f (b2 )
f (a )f (b) = f (b )f (a) = .
f (a) f (b)

It follows that there exists a positive rational number k such that

f (a2 ) = kf (a), for all a S. (1)

Substituting this into the functional equation yields

f (a)f (b)
f (ab) = , for all a, b S with a = b. (2)
k

Now combine the functional equation with (1) and (2) to obtain for all a S,

f (a)f (a5 ) f (a)f (a)f (a4 ) f (a)f (a)f (a2 )


f (a)f (a2 ) = f (a6 ) = = = .
k k2 k
It follows that f (a) = k for all a S.
Putting this into the functional equation yields k 2 = k. But 0, 1  S and hence
there is no solution. 

47

61
3. Solution 1 (Alexander Gunning, year 12, Glen Waverley Secondary College, VIC)
Answer: n = 60
Clearly all members of the sequence are positive rational numbers. For each non-
negative integer i let ai = pqii where pi and qi are positive integers with gcd(pi , qi ) = 1.
Therefore,
pi+1 2pi + qi pi+1 pi
= or = . (1)
qi+1 qi qi+1 pi + 2qi
If each of pi and qi are odd, then so are 2pi + qi , qi , pi , and pi + 2qi . Thus when the
RHSs of (1) are reduced to lowest terms, the numerators and denominators are still
odd. Hence pi+1 and qi+1 are odd. It follows inductively that if pi and qi are odd,
then pk and qk are odd for all k i. Since pqnn = 2014 we cannot have pi and qi both
being odd for any i n. Since gcd(pi , qi ) = 1, it follows that

pi and qi are of opposite parity for i = 0, 1, . . . , n. (2)

Suppose pi is odd for some i < n. We cannot have the second option in (1) because
that implies pi+1 and qi+1 are both odd, which contradicts (2). So we must have
the first option in (1), namely, pqi+1 i+1
= 2piq+q
i
i
. From (2), qi is even, and so we have
gcd(2pi + qi , qi ) = gcd(2pi , qi ) = 2. Hence
qi qi
pi odd pi+1 = pi + and qi+1 = for i < n. (3)
2 2

On the other hand, a similar argument shows that if pi is even for some i < n, then
we must take the second option in (1), namely pqi+1
i+1 pi
= pi +2q i
, and gcd(pi , pi +2qi ) = 1.
Hence

pi pi
pi even pi+1 = and qi+1 = + qi for i < n. (4)
2 2
In both (3) and (4), we have pi+1 + qi+1 = pi + qi . Since pn + qn = 2015, we have

pi + qi = 2015 for i 2015. (5)

If pi is odd, we may combine (3) and (5) to find


 qi qi 
(pi+1 , qi+1 ) = pi + ,
2 2
p q 
i i
, (mod 2015). (6)
2 2

A very similar calculation using (4) and (5) shows that (6) is also true if pi is even.
A simple induction on (6) yields,
p q0 
0
(pn , qn ) n, (mod 2015). (7)
2 2n

We require qn = 1. However, from (5), this is true if and only if

qn 1 (mod 2015).

48

62
Note that q0 = 1 because a0 is a positive integer. Thus from (7) we require

2n 1 (mod 2015).

Since 2015 = 3 13 31, we require 5, 13, 31 | 2n 1. Computing powers of 2, we see


that

5 | 2n 1 4 | n
13 | 2n 1 12 | n
31 | 2n 1 5 | n.

Since 60 = lcm(4, 12, 5) we conclude that 2015 | 2n 1 if and only if 60 | n. The


smallest such positive integer is n = 60. 

49

63
Solution 2 (Henry Yoo, year 12, Perth Modern School, WA)
Clearly all members of the sequence are positive rational numbers. For each positive
ai+1 1 2ai+1
integer i, we have ai = or ai = . Since ai > 0 we deduce that
2 1 ai+1

ai+1 1

if ai+1 > 1
2
ai = (1)

2ai+1
if ai+1 < 1.
1 ai+1
Thus ai is uniquely determined from ai+1 . Hence we may start from an = 2014 and
simply run the sequence backwards until we reach a positive integer.
From (1), if ai+1 = uv , then

uv

2v if u > v
ai =

2u
if u < v.
vu

Consequently, if we define the sequence of pairs of integers (u0 , v0 ), (u1 , v1 ), . . . by


(u0 , v0 ) = (2014, 1) and

(ui vi , 2vi ) if ui > vi
(ui+1 , vi+1 ) = (2)
(2ui , vi ui ) if ui < vi ,
ui
then ani = vi
for i = 0, 1, 2, . . ..
Observe from (2) that ui+1 + vi+1 = ui + vi . So since u0 = 2014 and v0 = 1, we have

ui + vi = 2015 for i = 0, 1, 2, . . .. (3)

Suppose that d is a common factor of ui+1 and vi+1 . Then (3) implies d | 2015, and
so d is odd. If ui > vi , then from (2) we have d | ui vi and d | 2vi . This easily
implies d | ui and d | vi . If ui < vi , we similarly conclude from (2) that d | ui and
d | vi . This inductively cascades back to give d | u0 and d | v0 . Since gcd(u0 , v0 ) = 1,
we deduce that d | 1. Therefore,

gcd(ui , vi ) = 1 for i = 0, 1, 2, . . .. (4)

From (3) and (4), we have ui = vi . Hence (2) yields ui+1 , vi+1 > 0, and so from (3)
we have
ui , vi {1, 2, . . . , 2014} for i = 0, 1, 2, . . .. (5)

Next we prove by induction that

(ui , vi ) (2i , 2i ) (mod 2015) for i = 0, 1, 2, . . .. (6)

The base case is immediate. Also, if (ui , vi ) (2i , 2i ) (mod 2015), then using (2)
we have

ui > vi (ui+1 , vi+1 ) (2i 2i , 2 2i ) (mod 2015)


i+1 i+1
(2 ,2 ) (mod 2015)

50

64
and
ui < vi (ui+1 , vi+1 ) (2(2i ), 2i (2i )) (mod 2015)
(2i+1 , 2i+1 ) (mod 2015).

Either way, this completes the inductive step.


We are given that a0 = uvnn is a positive integer. We know that un and vn are positive
integers with gcd(un , vn ) = 1. Thus we seek n such that vn = 1. From (5) and (6),
this occurs if and only if
2n 1 (mod 2015).
As in solution (1), the smallest such positive integer is n = 60. 

Comment 1 In this solution, if we were to run the sequence backwards from


an = 2014 until a positive integer is reached, the result would be
2014 2013 2011 2007 1999 1983 1951 1887 1759 1503 991 1982 1949 1883 1751 1487 959
1
, 2
, 4
, 8
, 16
, 32
, 64
, 128
, 256
, 512
, 1024
, 33
, 66
, 132
, 264
, 528
, 1056
,
1918 1821 1627 1239 463 926 1852 1689 1363 711 1422 829 1658 1301 587 1174 333
97
, 194
, 388
, 776
, 1552
, 1089
, 163
, 326
, 652
, 1304
, 593
, 1186
, 357
, 714
, 1428
, 841
, 1682
,
666 1332 649 1298 581 1162 309 618 1236 457 914 1828 1641 1267 519 1038 61
1349
, 683
, 1366
, 717
, 1434
, 853
, 1706
, 1397
, 779
, 1558
, 1101
, 187
, 374
, 748
, 1496
, 977
, 1954
,
122 244 488 976 1952 1889 1763 1511 1007 2014
1893
, 1771
, 1527
, 1039
, 63
, 126
, 252
, 504
, 1008
, 1
.
Since there are 61 terms in the above list, this also shows that n = 60.
Comment 2 A corollary of both solutions is that the only value of a0 which yields
a good sequence is a0 = 2014.

51

65
4. Solution 1 (Jeremy Yip, year 12, Trinity Grammar School, VIC)
Let be the maximal angle that occurs between a red line and a blue line. Let r
be a red line and b be a blue line such that the non-acute angle between them is .
Note that r and b divide the plane into four regions.
In the following diagrams, the two regions that lie within the angular areas defined
by are shaded. Also we orient our configuration so that the x-axis is the angle
bisector of r and b that passes through the two shaded regions. Let O be the
intersection of lines r and b.
Let  be another line in the configuration. Four options arise that need to be
considered.

2



1

In the first two diagrams,  does not pass through O. Observe that , r and b enclose
a triangle which is either completely shaded or completely unshaded. If the triangle
is completely shaded, as in the first diagram, then 1 = + > and 2 = + > .
But this is impossible because it implies that  cannot be blue or red. Hence the
triangle is completely unshaded, as in the second diagram.
In the last two diagrams,  passes through O. Observe that apart from the point O,
either all of  lies in the unshaded regions, or all of  lies in the shaded regions. If
 were to lie in the unshaded regions, as in the third diagram, then since + >
and + > , it would follow that  could not be red or blue. Hence  lies in the
shaded regions, as in the fourth diagram.
Let S be the set of intersection points of lines in the configuration that lie on r or b.
Consider any circle that lies to the right of all points of S and is tangent to r and
b in the right-hand shaded region. We claim that has the required property. It
suffices to show that every line in the configuration, apart from r and b, intersects
in two distinct points.

Let R and B be the points of tangency of with r and b, respectively, and let T be
the union of the segments OB and OR.

52

66
If  ( = r, b) is any line of the configuration, then the part of  lying in the right
shaded region is an infinite ray  .
Let F be the figure enclosed by T and the minor arc RB of . Then  passes into
the interior of F , and so intersects the boundary of F at least twice. Since  cannot
intersect T twice, it must intersect the minor arc RB.
Finally,  cannot be tangent to because that would imply that  intersects both
OB and OR (not at O). It follows that  intersects at two distinct points, as
desired. 

53

67
Solution 2 (Yang Song, year 12, James Ruse Agricultural High School, NSW)
We may rotate the plane so that no red line or blue line is vertical. Let 1 , 2 , . . . , 2n
be the lines listed in order of increasing gradient. Then there is a k such that lines
k and k+1 are oppositely coloured. By rotating our coordinate system and cyclicly
relabelling our lines we can ensure that 1 , 2 , . . . , 2n are listed in order of increasing
gradient, 1 and 2n are oppositely coloured, and no line is vertical. Without loss of
generality 1 is red and 2n is blue. Let O = 1 2n .
Let  (not one of the 2n lines) be a variable vertical line to the right of O. Let
R = 1  and B = 2n . Since the lines 2 , 3 , . . . , 2n1 have gradients lying
in between those of 1 and 2n , we can move  far enough to the right so that all
the intersection points of 2 , 3 , . . . , 2n1 with  lie between R and B. Let be the
excircle of ORB opposite A. We claim that has the required properties. To
prove this, it suffices to show that each line i (2 i 2n 1) intersects twice.

B

R

Let  be the vertical line which is tangent to and lying to the right of . Let
R =  1 and B  =  2n . Then is the incircle of BRR B  . The result now
follows because any line that intersects the opposite sides of a quadrilateral having
an incircle, must also intersect the incircle twice. 

54

68
5. Solution (APMO Problem Selection Committee)
There are two families of answers.

an = c(n + 2)n! for all n 1 and a0 = c + 1 for some integer c 2014.


an = c(n + 2)n! for all n 1 and a0 = c 1 for some integer c 2016.

Let a0 , a1 , a2 , . . . be a sequence of positive integers satisfying the given conditions.


We can rewrite (ii) as
sn+1 = (n + 1)an + hn ,
where hn {1, 1} . Substituting n with n 1 yields

sn = (n 1)an + hn1 ,

for n 2. Adding together the two equations we find

an+1 = (n + 1)an + nan1 + n (1)

for n 2 and where n {2, 0, 2}.


We also have |s2 2a1 | = 1, which yields a0 = 3a1 a2 1 3a1 , and therefore
a1 a30 671. Substituting n = 2 in (1), we find that a3 = 3a2 + 2a1 + 2 . Since
a1 | a3 , we have a1 | 3a2 + 2 , and therefore a2 223. Using (1), we obtain that
an 223 for all n 0.
Lemma 1 For n 4, we have an+2 = (n + 1)(n + 4)an .
Proof For n 3 we have

an = nan1 + (n 1)an2 + n1
> nan1 + 3. (2)

By applying (1) and (2) with n substituted by n 1, we have for n 4,

an = nan1 + (n 1)an2 + n1
< nan1 + (an1 3) + n1
< (n + 1)an1 . (3)

Using (1) to write an+2 in terms of an and an1 along with (2), we have for n 3,

an+2 = (n + 3)(n + 1)an + (n + 2)nan1 + (n + 2)n + n+1


< (n + 3)(n + 1)an + (n + 2)nan1 + 3(n + 2)
< (n2 + 5n + 5)an .

Also for n 4,

an+2 = (n + 3)(n + 1)an + (n + 2)nan1 + (n + 2)n + n+1


> (n + 3)(n + 1)an + nan
= (n2 + 5n + 3)an .

Since an | an+2 , we have an+2 = (n2 + 5n + 4)an = (n + 1)(n + 4)an , as desired. 


(n+1)(n+3)
Lemma 2 For n 4, we have an+1 = n+2
an .

55

69
Proof Using the recurrence an+3 = (n + 3)an+2 + (n + 2)an+1 + n+2 and writing
an+3 , an+2 in terms of an+1 , an according to lemma 1 we obtain

(n + 2)(n + 4)an+1 = (n + 3)(n + 1)(n + 4)an + n+2 .

Hence n + 4 | n+2 , which yields n+2 = 0 and an+1 = (n+1)(n+3)


n+2
an , as desired. 
(n+1)(n+3)
Lemma 3 For n 1, we have an+1 = n+2
an .
Proof Suppose there exists n 1 such that an+1 = (n+1)(n+3)
n+2
an . By lemma 2, there
is a greatest integer 1 m 3 with this property. Then am+2 = (m+2)(m+4)
m+3
am+1 .
(m+1)(m+3)
If m+1 = 0, then am+1 = m+2
am , which contradicts our choice of m. Thus
m+1 = 0.
Clearly m + 3 | am+1 . So we have am+1 = (m + 3)k and am+2 = (m + 2)(m + 4)k,
for some positive integer k. Then

(m + 1)am + m+1 = am+2 (m + 2)am+1 = (m + 2)k.

So, am | (m + 2)k m+1 . But am also divides am+2 = (m + 2)(m + 4)k. Combining
the two divisibility conditions, we obtain am | (m + 4)m+1 .
Since m+1 = 0, we have am | 2m + 8 14, which contradicts the previous result
that an 223 for all non-negative integers n. 
So, an+1 = (n+1)(n+3)
n+2
an for n 1. Substituting n = 1 yields 3 | a1 . Letting a1 = 3c,
we have by induction that an = n!(n + 2)c for n 1. Since |s2 2a1 | = 1, we then
get a0 = c 1, yielding the two families of solutions.
Finally, since (n+2)n! = n!+(n+1)!, it follows that sn+1 = c(n+2)!+(1)n (ca0 ).
Hence both families of solutions satisfy the given conditions. 

56

70
27TH ASIAN PACIFIC MATHEMATICS OLYMPIAD RESULTS
Top 10 Australian scores

Name School Year Total Award

Jeremy Yip Trinity Grammar School VIC 12 28 Gold

Alexander Gunning Glen Waverley Secondary College VIC 12 28 Silver

Yang Song James Ruse Agricultural High School NSW 12 23 Silver

Henry Yoo Perth Modern School WA 12 20 Bronze

Thomas Baker Scotch College VIC 11 20 Bronze

Allen Lu Sydney Grammar School NSW 12 20 Bronze

Ilia Kucherov Westall Secondary College VIC 11 19 Bronze

Seyoon Ragavan Knox Grammar School NSW 11 17

Yong See Foo Nossal High School VIC 11 16

Alan Guo Penleigh and Essendon Grammar School VIC 12 14

Country scores

Number of
Rank Country Score Gold Silver Bronze Hon.Men
Contestants
1 USA 10 298 1 2 4 3

2 Korea 10 279 1 2 4 3

3 Russia 10 266 1 2 4 3

4 Singapore 10 259 1 2 4 3

5 Japan 10 256 1 2 4 3

6 Canada 10 237 1 2 4 3

7 Thailand 10 228 1 2 4 3

8 Taiwan 10 222 1 2 4 3

9 Australia 10 205 1 2 4 3

10 Brazil 10 202 1 2 4 3

11 Peru 10 185 0 3 4 3

12 Mexico 10 169 1 1 5 3

13 Hong Kong 10 167 0 3 4 3

14 Kazakhstan 10 163 0 2 5 3

15 Indonesia 10 161 0 3 4 3

16 Malaysia 10 134 0 3 3 3

17 India 10 127 0 1 5 3

71
Number of
Rank Country Score Gold Silver Bronze Hon.Men
Contestants
17 Tajikistan 10 127 0 0 6 4

19 Bangladesh 10 122 0 0 7 1

20 Philippines 10 105 0 3 0 2

21 Turkmenistan 10 99 0 0 4 3

22 Saudi Arabia 10 94 0 0 4 1

23 New Zealand 10 86 0 2 1 1

24 Argentina 10 73 0 0 1 3

24 Colombia 10 73 0 1 3 1

26 Syria 6 52 0 0 1 5

27 Sri Lanka 7 48 0 0 1 4

28 El Salvador 7 47 0 0 2 2
Trinidad and
29 10 31 0 0 1 0
Tobago
30 Ecuador 10 27 0 0 1 1

31 Costa Rica 5 20 0 0 0 1

32 Panama 2 12 0 0 0 0

33 Cambodia 2 9 0 0 0 1

Total 299 4583 11 42 102 81

72
AMOC SELECTION
2015 IMO Team Selection School SCHOOL
The 2015 IMO Selection School was held 514 April at Robert Menzies College, Macquarie
University, Sydney. The main qualifying exams are the AMO and the APMO from which
25 students are selected for the school.
The routine is similar to that for the December School of Excellence; however, there is
the added interest of the actual selection of the Australian IMO team. This year the IMO
would be held in Chiang Mai, Thailand.
The students are divided into a junior group and a senior group. This year there were 10
juniors and 15 seniors. It is from the seniors that the team of six for the IMO plus one
reserve team member is selected. The AMO, the APMO and the final three senior exams
at the school are the official selection criteria.
My thanks go to Andrew Elvey Price, Ivan Guo, Victor Khou, and Konrad Pilch, who
assisted me as live-in staff members. Also to Peter Brown, Vaishnavi Calisa, Mike Clapper,
Nancy Fu, Declan Gorey, David Hunt, Vickie Lee, Peter McNamara, John Papantoniou,
Christopher Ryba, Andy Tran, Gareth White, Rachel Wong, Sampson Wong, Jonathan
Zheng, and Damon Zhong, all of whom came in to give lectures or help with the marking
of exams.

Angelo Di Pasquale
Director of Training, AMOC

2015 Australian IMO team

Name Year School State


Alexander Gunning 12 Glen Waverley Secondary College VIC
Ilia Kucherov 11 Westall Secondary College VIC
Seyoon Ragavan 11 Knox Grammar School NSW
Yang Song 12 James Ruse Agricultural High School NSW
Kevin Xian 11 James Ruse Agricultural High School NSW
Jeremy Yip 12 Trinity Grammar School VIC
Reserve
Yong See Foo 11 Nossal High School VIC

73
2015 Australian IMO Team

Name School Year

Alexander Gunning Glen Waverley Secondary College VIC 12

Ilia Kucherov Westall Secondary College VIC 11

Seyoon Ragavan Knox Grammar School NSW 11

Yang Song James Ruse Agricultural High School NSW 12

Kevin Xian James Ruse Agricultural High School NSW 11

Jeremy Yip Trinity Grammar School VIC 12

Reserve

Yong See Foo Nossal High School VIC 11

2015 Australian IMO Team, from left, Jeremy Yip, Alexander Gunning, Yang Song, Kevin Xian, Ilia
Kucherov and Seyoon Ragavan.

74
Participants at the 2015 IMO Selection School

Name School Year

Seniors

Thomas Baker Scotch College VIC 11

Matthew Cheah Penleigh and Essendon Grammar School VIC 10

Michelle Chen Methodist Ladies' College VIC 11

Yong See Foo Nossal High School VIC 11

Alexander Gunning Glen Waverley Secondary College VIC 12

Alan Guo Penleigh and Essendon Grammar School VIC 12

Ilia Kucherov Westall Secondary College VIC 11

Leo Li Christ Church Grammar School WA 11

Allen Lu Sydney Grammar School NSW 12

Seyoon Ragavan Knox Grammar School NSW 11

Kevin Xian James Ruse Agricultural High School NSW 11

Jeremy Yip Trinity Grammar School VIC 12

Henry Yoo Perth Modern School WA 12

Wilson Zhao Killara High School NSW 11

Juniors

Bobby Dey James Ruse Agricultural High School NSW 10

Rachel Hauenschild Kenmore State High School QLD 10

William Hu Christ Church Grammar School WA 9

Tony Jiang Scotch College VIC 10

Charles Li Camberwell Grammar School VIC 9

Jack Liu Brighton Grammar School VIC 9

Isabel Longbottom Rossmoyne Senior High School WA 10

Hilton Nguyen Sydney Technical High School NSW 9

Tommy Wei Scotch College VIC 9

Wen Zhang St Joseph's College, Gregory Terrace QLD 9

75
IMO Team IMO TEAM School
Preparation PREPARATION SCHOOL
The pre-IMO July school is always a great reality check when it comes to our perception
of the teams ability. This is of course because we train with the UK team. Our joint
training school was held 28 July at Nexus International School, Singapore.
The routine for the teams each day consisted of an IMO trial exam in the morning, free
time in the afternoon while their papers were being assessed, a short debrief of the exam
late in the afternoon followed by going out to dinner each evening. The results of training
showed that both teams were quite strong, with the UK having the edge.
The final exam also doubles as the annual Mathematics Ashes contest. Australia won
the Ashes in its inaugural year, lost them the next year, and have not been able to win
them back since. There have been a few close calls, and even a tie in 2011. In another
heart-breaking nail biter, the UK again retained the Ashes after both teams tied on 84
points apiece.

Angelo Di Pasquale
IMO Team Leader

76
THE MATHEMATICS
The 2015 Mathematical ASHES
Ashes: AUS v UK

Exam

1. Does there exist a 2015 2015 array of distinct positive integers such that the sums
of the entries on each row and on each column yield 4030 distinct perfect squares?

2. Let and O be the circumcircle and the circumcentre of an acute-angled triangle


ABC with AB > BC. The angle bisector of ABC intersects at M = B. Let
be the circle with diameter BM . The angle bisectors of AOB and BOC intersect
at points P and Q, respectively. The point R is chosen on the line P Q so that
BR = M R.
Prove that BR  AC.
(In this problem, we assume that an angle bisector is a ray.)

3. We are given an infinite deck of cards, each with a real number on it. For every
real number x, there is exactly one card in the deck that has x written on it. Now
two players draw disjoint sets A and B of 100 cards each from this deck. We would
like to define a rule that declares one of them a winner. This rule should satisfy the
following conditions:

1. The winner only depends on the relative order of the 200 cards: if the cards are
laid down in increasing order face down and we are told which card belongs to
which player, but not what numbers are written on them, we can still decide
the winner.
2. If we write the elements of both sets in increasing order as A = {a1 , a2 , . . . , a100 }
and B = {b1 , b2 , . . . , b100 }, and ai > bi for all i, then A beats B.
3. If three players draw three disjoint sets A, B, C from the deck, A beats B and
B beats C, then A also beats C.

How many ways are there to define such a rule?


(In this problem, we consider two rules as different if there exist two sets A and B
such that A beats B according to one rule, but B beats A according to the other.)

Results

Q1 Q2 Q3 Q1 Q2 Q3
AUS 1 7 7 7 21 UNK 1 7 7 5 19
AUS 2 7 0 7 14 UNK 2 7 2 0 9
AUS 3 7 7 0 14 UNK 3 5 0 0 5
AUS 4 7 5 0 12 UNK 4 3 7 7 17
AUS 5 3 6 1 10 UNK 5 7 6 7 20
AUS 6 6 7 0 13 UNK 6 7 7 0 14
37 32 15 84 36 29 19 84

77
THE MATHEMATICS ASHES RESULTS
The 8th Mathematics Ashes competition at the joint pre-IMO training camp in Putrajaya, was tied; the results for
the two teams were as follows, with each team scoring a total of 84:

Australia

Code Name Q1 Q2 Q3 Total

AUS1 Alexander Gunning 7 7 7 21

AUS2 Ilia Kucherov 7 0 7 14

AUS3 Seyoon Ragavan 7 7 0 14

AUS4 Yang Song 7 5 0 12

AUS5 Kevin Xian 3 6 1 10

AUS6 Jeremy Yip 6 7 0 13

TOTAL 37 32 15 84

United Kingdom

Code Name Q1 Q2 Q3 Total

UNK1 Joe Benton 7 7 5 19

UNK2 Lawrence Hollom 7 2 0 9

UNK3 Samuel Kittle 5 0 0 5

UNK4 Warren Li 3 7 7 17

UNK5 Neel Nanda 7 6 7 20

UNK6 Harvey Yau 7 7 0 14

TOTAL 36 29 19 84

78
The 56th International Mathematical Olympiad, Chiang Mai,
IMO TEAM LEADERS
Thailand REPORT

The 56th International Mathematical Olympiad (IMO) was held 416 July in Chiang Mai,
Thailand.
This was the largest IMO in history with a record number of 577 high school students
from 104 countries participating. Of these, 52 were girls.
Each participating country may send a team of up to six students, a Team Leader and a
Deputy Team Leader. At the IMO the Team Leaders, as an international collective, form
what is called the Jury. This Jury was chaired by Soontorn Oraintara.
The first major task facing the Jury is to set the two competition papers. During this
period the Leaders and their observers are trusted to keep all information about the contest
problems completely confidential. The local Problem Selection Committee had already
shortlisted 29 problems from 155 problem proposals submitted by 53 of the participating
countries from around the world. During the Jury meetings one of the shortlisted problems
had to be discarded from consideration due to being too similar to material already in
the public domain. Eventually, the Jury finalised the exam questions and then made
translations into the more than 50 languages required by the contestants. Unfortunately,
due to an accidental security breach, the second days paper had to be changed on the
night before that exam was scheduled. This probably resulted in a harder than intended
second day.
The six questions that ultimately appeared on the IMO contest are described as follows.

1. A relatively easy two-part problem in combinatorial geometry proposed by the


Netherlands. It concerns finite sets of points in the plane in which the perpen-
dicular bisector of any pair of points in such a set also contains another point of the
set.

2. A medium classical number theory problem proposed by Serbia.

3. A difficult classical geometry problem in which one is asked to prove that a certain
two circles are mutually tangent. It was proposed by Ukraine.

4. A relatively easy classical geometry problem proposed by Greece.

5. A medium to difficult functional equation proposed by Albania.

6. A difficult problem in which one is asked to prove an inequality about a sequence


of integers. Although it does not seem so at first sight, the problem is much more
combinatorial than algebraic. It was inspired by a notation used to describe juggling.
The problem was proposed by Australia.

These six questions were posed in two exam papers held on Friday 10 July and Saturday
11 July. Each paper had three problems. The contestants worked individually. They were
allowed four and a half hours per paper to write their attempted proofs. Each problem
was scored out of a maximum of seven points.
For many years now there has been an opening ceremony prior to the first day of compe-
tition. HRH Crown Princess Sirindhorn presided over the opening ceremony. Following
the formal speeches there was the parade of the teams and the 2015 IMO was declared
open.

79
After the exams the Leaders and their Deputies spent about two days assessing the work
of the students from their own countries, guided by marking schemes, which had been
discussed earlier. A local team of markers called Coordinators also assessed the papers.
They too were guided by the marking schemes but are allowed some flexibility if, for
example, a Leader brought something to their attention in a contestants exam script
that is not covered by the marking scheme. The Team Leader and Coordinators have to
agree on scores for each student of the Leaders country in order to finalise scores. Any
disagreements that cannot be resolved in this way are ultimately referred to the Jury.
The IMO paper turned out to be quite difficult. While the easier problems 1 and 4
were quite accessible, the other four problems 2, 3, 5 and 6 were found to be the most
difficult combination of medium and difficult problems ever seen at the IMO. There were
only around 30 complete solutions to each of problems 2, 3 and 5. Problem 6 was very
difficult, averaging just 0.4 points. Only 11 students scored full marks on it.
The medal cuts were set at 26 for gold, 19 for silver and 14 for bronze.11 Consequently,
there were 282 (=48.9%) medals awarded. The medal distributions22 were 39 (=6.8%)
gold, 100 (=17.3%) silver and 143 (=24.8%) bronze. These awards were presented at the
closing ceremony. Of those who did not get a medal, a further 126 contestants received
an honourable mention for solving at least one question perfectly.
Alex Song of Canada was the sole contestant who achieved the most excellent feat of
a perfect score of 42. He now leads the IMO hall of fame, being the most decorated
contestant in IMO history. He is the only person to have won five IMO gold medals.33 He
was given a standing ovation during the presentation of medals at the closing ceremony.
Congratulations to the Australian IMO team on an absolutely spectacular performance
this year. They smashed our record rank44 to come 6th, and they also smashed our record
medal haul, bringing home two Gold and four Silver medals.55 This is the first time that
each team member has achieved Silver or better. The team finished ahead of many of
the traditionally stronger teams. In particular, they finished ahead of Russia, whom we
would have considered as untouchable.
Congratulations to Gold medallist Alexander Gunning, year 12, Glen Waverley Secondary
College. He is now the most decorated Australian at the IMO, being the only Australian
to have won three Gold medals at the IMO. On each of these occasions he also finished
in the top 10 in individual rankings.66 He is now equal 17th on the IMOs all-time hall of
fame.
Congratulations also to Gold medallist Seyoon Ragavan, year 11, Knox Grammar School.
Seyoon solved four problems perfectly and was comfortably above the Gold medal cut.
He was individually ranked 19th.
11
This was the lowest ever cut for gold, and the equal lowest ever cut for silver. (This was indicative of
the difficulty of the exam, not the standard of the contestants.)
2 The total number of medals must be approved by the Jury and should not normally exceed half the
2

total number of contestants. The numbers of gold, silver and bronze medals must be approximately in
the ratio 1:2:3.
3
3
In his six appearances at the IMO, Alex Song won a bronze medal in 2010, and followed up with gold
medals in 2011, 2012, 2013, 2014 and 2015.
4 The ranking of countries is not officially part of the IMO general regulations. However, countries are
4

ranked each year on the IMOs official website according to the sum of the individual student scores
from each country.
5
5
Australias best performance prior to this was the dream team of 1997. They came 9th, with a medal
tally of two Gold, three Silver and one Bronze.
6
6
In his four appearances at the IMO, Alexander won a bronze medal in 2012, and followed up with gold
medals in 2013 (8th), 2014 (1st) and 2015 (4th).

80
And congratulations to our four Silver medallists: Ilia Kucherov, year 11, Westall Sec-
ondary College; Yang Song, year 12, James Ruse Agricultural High School; Kevin Xian,
year 11, James Ruse Agricultural High School; and Jeremy Yip, year 12, Trinity Grammar
School.
Three members of this years team are eligible for selection to the 2016 IMO team. So
while it is unlikely we will be able to repeat this years stellar performance, the outlook
seems promising.
Congratulations also to Ross Atkins and Ivan Guo, who were IMO medallists with the
Australian team when they were students.77 They were the authors of the juggling-inspired
IMO problem number six. In fact Ross is a proficient juggler.
The 2015 IMO was organised by: The Institute for the Promotion of Teaching Science
and Technology, Chiang Mai University, The Mathematical Association of Thailand under
the Patronage of His Majesty the King, and The Promotion of Academic Olympiad and
Development of Science Education Foundation.
The 2016 IMO is scheduled to be held July 6-16 in Hong Kong. Venues for future IMOs
have been secured up to 2019 as follows.
2017 Brazil
2018 Romania
2019 United Kingdom
Much of the statistical information found in this report can also be found at the official
website of the IMO.
www.imo-official.org

Angelo Di Pasquale
IMO Team Leader, Australia

7
7
Ross and Ivan won Bronze at the 2003 IMO, and Ivan won Gold at the 2004 IMO.

81
Ross Atkins demonstrates his juggling skills. (Photo credit: Gillian Bolsover)

84

82
INTERNATIONAL MATHEMATICAL OLYMPIAD

IMO Papers
Language: English
Day: 1

Friday, July 10, 2015Day: 1


IMO Papers
Problem 1. We say that a finite set S of points in the plane is balanced if, for any two
different points A and B in S, there is a point C in S such that AC = BC. We say that
S is centre-free if for any three different points A, B and C in S, there is no point Day:P 1in
S such that P A = P B = P C.
Friday, July 10, 2015
(a) Show that for all integers n 3, there exists a balanced set consisting of n points.
Problem 1. We say that a finite S set S of points in the plane is balanced if, for any two
A (b) Determine
S Aalland
B points
different integers n C
B in S, 3 for
thereSis which
a pointthere
in exists
CAC = sucha that
S BC balanced
AC =centre-free
S We say
BC. set that
con-
sisting of n points.B C S P S P A = P B =
S is centre-free if for any three different points A, B and C in S, there is no point PP Cin
A
S such that P A = P B = P C.
n3 n
Problem 2. Determine all triples (a, b, c) of positive integers such that each of the
(a) Show that for all
numbers n integers
3 n 3, there exists a balanced set consisting of n points.n

(b) Determine all integers n ab3for bc there


c, which a, ca b a balanced centre-free set con-
exists
sisting
is a power of of
2. n points. (a, b, c)
(A power of 2 is an integer of the form 2n , where n is a non-negative integer.)
Problem 2. Determine allabtriples c, bc (a,b,a,c) of
ca positive
b integers such that each of the
Problem
numbers 3. Let ABC be an acute triangle with AB > AC. Let be its circumcircle,
2
H its orthocentre, and F the foot abof the
c, altitude
bc a, fromca A.b Let M be the midpoint of BC.
Let Q be the point on such that HQA = 90 , and let K be the point on such that
2 2 n
n
is a power of 2.
HKQ = 90 . Assume that the points A, B, C, K and Q are all different, and lie on
(A power
in this order.2 is an integer of the form 2n , where
of ABC AB >nAC is a non-negative
integer.) H
F A M BC Q
Prove that the circumcircles of triangles KQH and F KM are tangent to each other.

Problem 3. HQA = 90be
Let ABC an acute K triangle with AB > AC. LetHKQ = circumcircle,
be its 90
A B C K Q
H its orthocentre, and F the foot of the altitude from A. Let M be the midpoint of BC.
Let Q be the point on such that KQH HQA F =KM 90 , and let K be the point on such that
HKQ = 90 . Assume that the points A, B, C, K and Q are all different, and lie on
in this order.
Prove that the circumcircles of triangles KQH and F KM are tangent to each other.

Language: English Time: 4 hours and 30 minutes


Each problem is worth 7 points
83
Language: English

Day: Day:
2 2
Saturday, July 11, 2015
Day: 2
Problem 4. Triangle ABC has circumcircle and circumcentre O. A circle with
centre A intersects the segment BC at points D and E, such thatSaturday, B, D, E July
and C11,are all
2015
different and lie on line BC in this order. Let F and G be the points of intersection of
and , such that A, F , B, C and G lie on in this order.OLet K be the second point
Problem 4. TriangleABC ABC has circumcircle and circumcentre O.
A circle with A
of intersection ofBCthe circumcircle
D BCofattriangle BDFB and
E the
E segment
D such AB. Let L be the
C B, D,
centre A intersects the segment points D and E, that E and C are all
second
BC point of intersection of the circumcircle of triangle CGE and the segment CA.
different and lie on lineFBC inGthis order. Let F and G be thepointsof intersection A F of
B C Suppose
G
that the lines F K and K
GL are different and intersect at the point X. Prove
and , such that A, F , B, C and G lie on in this order. Let K be the second point that
ofBDF
X lies on the line
intersection AB
AO.circumcircle
of the L of triangle BDF and the segment AB. Let L be the
CGE point of intersection
second CA of the circumcircle of triangle CGE and the segment CA.
Problem 5. Let R denote the set of real numbers. Determine all functions f :XR R
Suppose that theF K lines F GL
K and GL are different and intersect at the X point X. Prove that
satisfying
AO the equation
X lies on the line AO.
f (x + f (x + y)) + f (xy) = x + f (x + y) + yf (x)
Problem 5.R Let R denote the set of real numbers. Determine fall: R R f: R R
functions
satisfying
for all realthe equation
numbers x and y. 
f x + f (x + y) + f (xy) = x + f (x + y) + yf (x)
f (x + f (xa+, ay)), .+
Problem 6. x The ysequence . . fof(xy) = x +satisfies
integers f (x + y) + following
the yf (x) conditions:
1 2

for all real numbers x and y.


a1 ,all
(i) 1 aj 2015 for a2 ,j. .
. 1;

 aj k+2015
1Problem
(ii) a6. 1
The jsequence a1 , a2 , . . . of integers satisfies the following conditions:
k = + a for all 1 k < .

ak =
k +(i) + a 1 all
k< j 1;
Prove1that ajthere
2015 for
exist two positive integers b and N such that
b N 
(ii) k + ak = + a for all 1 k <n . 
 n   
 
 
(aj  b)
 
10072
 j=m+1
Prove that there exist two positive (aj b)b and
integers 1007 2
N such that
 
 n
j=m+1 
for all integers m and n satisfying n > m N .
 
m n 
n > m  N (aj b) 10072
 
j=m+1

for all integers m and n satisfying n > m N .

Language: English Time: 4 hours and 30 minutes


Each problem is worth 7 points

84
Language: English Time: 4 hours and 30 minutes
INTERNATIONAL MATHEMATICAL OLYMPIAD
SOLUTIONS
1. Solution (All members of the 2015 Australian IMO team solved this problem using
similar methods. Here we present the solution by Yang Song, year 12, James Ruse
Agricultural High School, NSW. Yang was Silver medallist with the 2015 Australian
IMO team.)

(a) For n odd, we may take S to be the set of vertices of a regular n-gon P.
It seems obvious that S is balanced but we shall prove it anyway. Let A and
B be any two vertices of P. Since n is odd, one side of the line AB contains an
odd number of vertices of P. Thus if we enumerate the vertices of P in order
from A around to B on that side of AB, one of them will be the middle one,
and hence be equidistant from A and B.
For n even, say n = 2k we may take S to be the set of vertices of a collection
of k unit equilateral triangles, all of which have a common vertex, O say, and
exactly one pair of them has a second common vertex. Note that apart from
O, all of the vertices lie on the unit circle centred at O.
The reason why this works is as follows. Let A and B be any two points in S.
If they are both on the circumference of the circle, then OA = OB and O S.
If one of them is not on the circumference, say B = O, then by construction
there is a third point C S such that ABC is equilateral, and so AC = BC.
The cases for n = 11 and n = 12 are illustrated below.

(b) We claim that a balanced centre-free set of n points exists if and only if n is
odd.
Note that the construction used in the solution to part (a) is centre-free. We
shall show that there is no balanced centre-free set of n points if n is even.
For any three points A, X, Y S, let us write A {X, Y } to mean AX = AY .
We shall estimate the number of instances of A {X, Y } in two different ways.
First, note that if A {X, Y } and A {X, Z} where Y = Z, then S cannot
be centre-free
 n1  because AX = AY = AZ. Hence for a given point A, there are
at most 2 pairs {X, Y } such that A {X, Y }. Since there are  n choices
for A, the total number of instances of A {X, Y } is at most n n1 2
.
On the other hand, since S is balanced, for each pair of points X, Y S, there
is at leastone
n
 point A such that A {X, Y }. Since the number ofnpairs 
{X, Y } is 2 , the total number of instances of A {X, Y } is at least 2 .
   
If we combine our estimates, we obtain n n1
 n1 2
n2 , which simplifies to
2
n1
2
. This final inequality is impossible if n is even. 

85
58
Comment 1 A nice graph theoretical interpretation of the solution to part (b) was
given by Kevin Xian, year 11, James Ruse Agricultural High School, NSW. Kevin
was a Silver medallist with the 2015 Australian IMO team.
Let S  denote the set of unordered pairs of elements of S. Form a directed bipartite
graph G as follows. The vertex set of G is S S  . The directed edges of G are
simply all the instances of A {X, Y} asper the solution to part (b) above. Then
n1
the
n
 total outdegree of G is at most n 2 , while the total indegree of G is at least
2
. The inequality found in the solution to part (b) is simply a consequence of the
total indegree being equal to the total outdegree.

Comment 2 Alternative constructions for n odd in part (a) were found by Yang
Song, year 12, James Ruse Agricultural High School, NSW, and Jeremy Yip, year
12, Trinity Grammar School, VIC. Yang and Jeremy were Silver medallists with the
2015 Australian IMO team.
For n = 2k + 1, let S be the set of vertices of a collection of k unit equilateral
triangles, all of which have exactly a common vertex, O say, and no two of which
have any other common vertices besides O. Note that apart from O, all of the
vertices lie on the unit circle centred at O. The case for n = 13 is illustrated below.

Comment 3 At the time of writing, the classification of balanced sets is an open


problem. From the foregoing, we see that for n odd there are two infinite families of
balanced sets, and for n even there is one infinite family. It is unknown if there are
any other infinite families which do not fit into this scheme. Note however, there
are sporadic constructions which do not fit into either of these families. Some of
these are shown below. (All drawn segments are of unit length.)

59

86
2. Solution 1 (Jeremy Yip, year 12, Trinity Grammar School, VIC. Jeremy was a
Silver medallist with the 2015 Australian IMO team.)
The answers are (a, b, c) = (2, 2, 2), (2, 2, 3), (2, 6, 11), (3, 5, 7) and their permuta-
tions. It is straightforward to verify that they all work.
Note that ab c = bc a (b + 1)(a c) = 0 a = c.
We divide our solution into five cases as follows.
Case 1 At least two of a, b, c are equal.
Case 2 All of a, b, c are different and none of them are even.
Case 3 All of a, b, c are different and one of them is even.
Case 4 All of a, b, c are different and two of them are even.
Case 5 All of a, b, c are different and all three of them are even.
Case 1 Without loss of generality a = b. Then b2 c and bc b are powers of 2.
That is,

b2 c = 2x (1)
b(c 1) = 2y (2)

for some non-negative integers x and y. Equation (2) implies that b = 2p and
c 1 = 2q for some non-negative integers p and q. Putting these into (1) we find

22p = 2x + 2q + 1. (3)

Since RHS(3) 3, we have p 1, and so 4 | 2x + 2q + 1.


If q = 0, then (3) becomes 22p 2x = 2. The only powers of two that differ by 2 are
2 and 4. Hence x = p = 1, which quickly leads to (a, b, c) = (2, 2, 2).
Similarly, if x = 0, then (3) becomes 22p 2q = 2. Hence q = p = 1, and so
(a, b, c) = (2, 2, 3).
Finally, if q, x 1, then (3) cannot hold because the LHS is even while the RHS is
odd.
Case 2 Without loss of generality we may suppose that a > b > c, where a, b, c
are all odd.
It follows that ab c > ac b and a 3. Since ab c and ac b are powers of two,
the smaller divides the larger. Therefore,

ab c 0 (mod ac b)
a(ac) c 0 (mod ac b)
(a 1)(a + 1) 0 (mod ac b). (c is odd)

Since a 1 and a + 1 are consecutive even positive integers, exactly one of them is
divisible by 4, while the other is even but not divisible by 4. Since ac b is a power
of two, it follows that either ac b | 2(a 1) or ac b | 2(a + 1). Either way, we
have
ac b 2(a + 1).

Since a > b, we have ac a < 2(a + 1), which can be rearranged as

(c 3)a < 2. (4)

60

87
Observe that a 5, b 3 and c 1. This is because a, b, c are all odd and
a > b > c 1.
If c > 3 then (4) implies a < 2. This contradicts a 5.
If c = 1, then a b and b a are powers of two, which is impossible because they
sum to zero.
If c = 3, then 3b a and 3a b are powers of two. Since 3b a < 3a b, we have

3a b 0 (mod 3b a)
3(3b) b 0 (mod 3b a)
8 0 (mod 3b a). (b is odd)

Hence 3b a {1, 2, 4, 8}.


Certainly 3b a = 1, because a and b are odd.
If 3b a = 2, then a = 3b 2, and so 3a b = 8b 6 2 (mod 4). Hence 8b 6 = 2,
and b = 1. But this impossible because b > c 1.
If 3b a = 4, then a = 3b 4, and so 3a b = 8b 12 4 (mod 8). Hence
8b 12 = 4, and b = 2, which contradicts that b is odd.
If 3b a = 8, then a = 3b 8, and so 3a b = 8(b 3). Hence b 3 is a power of
two. Thus b = 3 + 2k for some positive integer k. Also

ab c = b(3b 8) 3
= (3b + 1)(b 3)

is a power of two. Hence 3b+1 = 32k +10 is also a power of two. This is impossible
for k 2 because 3 2k + 10 2 (mod 4) and 3 2k + 10 = 2. However, for k = 1,
we find 3b + 1 = 16. Thus b = 5 and a = 3b 8 = 7, and we have found the solution
(a, b, c) = (3, 5, 7).
Case 3 Without loss of generality a is even, while b and c are odd.
Then ab c and ac b are both odd. Hence ab c = ac b = 1. This implies b = c,
which is a contradiction. So this case does not occur.
Case 4 Without loss of generality c is odd, while a and b are even with a > b.
This immediately implies that a 4. Also, since ab c is odd, we have ab c = 1.
Thus c = ab 1 3 (mod 4). Hence c 3.
Let 2k (note k 1) be the greatest power of two dividing both a and b. Then
a = 2k m and b = 2k n for some integers m > n 1.
We have ac b = 2k (mc n). Thus mc n is a power of two. Thus m and n are of
the same parity because otherwise mc n would be an odd number that is greater
than 1. From the choice of k, this implies that m and n are both odd.
Since a > b we have bc a < ac b. Because they are both powers of two, the
smaller divides the larger. Therefore,

ac b 0 (mod bc a) (5)
(bc)c b 0 (mod bc a)
2k n(c2 1) 0 (mod 2k (nc m))
(c 1)(c + 1) 0 (mod nc m). (n is odd) (6)

61

88
Using similar reasoning as in case 2, since nc m is a power of two, and c 1 and
c + 1 are consecutive even positive integers, we deduce that

nc m 2(c + 1)
n(4 mn 1) m 2 4k mn (c = ab 1 = 4k mn 1)
k

n
4k (n2 2n)
m
< 1. (m > n)

Since n is odd, this implies n = 1. Putting this into (6), we find

(c 1)(c + 1) 0 (mod c m)
(m 1)(m + 1) 0 (mod c m).

Using similar reasoning as in case 2, since c m is a power of two, and m 1 and


m + 1 are consecutive even positive integers, we deduce that

c m 2(m + 1)
4k m m 2m + 2 (c=4k m 1)
4k m < 3m + 2
k = 1.

To summarise, we have found (a, b, c) = (2m, 2, 4m 1). In particular, c = 2a 1.


Putting this into (5) we find,

a(2a 1) 2 0 (mod 3a 2)
3a(6a 3) 18 (mod 3a 2)
2 18 (mod 3a 2).

Hence 3a 2 | 16. Since a 4, we have 3a 2 = 16. Thus a = 6, and so


(a, b, c) = (6, 2, 11).
Case 5 Without loss of generality we may suppose that a = 2 A, b = 2 B and
c = 2 C, where 1, and A, B and C are odd positive integers. It
follows that

ab c = 2 (2+ AB C) and ac b = 2 (2+ AC B).

Since 1, we have that 2+ AB C and 2+ AC B are odd.


But since ab c and ac b are powers of two, it follows that

ab c = 2 and ac b = 2 .

Since 2 | c, we have 2 c, and similarly 2 b. Hence

ab 2c and ac 2b. (7)

Multiplying these inequalities together yields

a2 bc 4bc.

Hence a 2, and thus a = 2 because a is even. But now the two inequalities in
(7) become 2b 2c and 2c 2b. Hence b = c, which contradicts that a, b, c are all
different. So this case does not occur, and the proof is complete. 

62

89
Solution 2 (Alex Gunning, year 12, Glen Waverley Secondary College, VIC. Alex
was a Gold medallist with the 2015 Australian IMO team.)
The case where two of the variables are equal is handled in the same way as in
solution 1.
Without loss of generality we may assume that a > b > c. If c = 1, then we would
require both a b and b a to be powers of two. But this is impossible because
their sum is zero.
Hence a > b > c 2.
It easily follows that ab c ca b bc a. Since a smaller power of two always
divides a larger power of two, we have

bc a | ca b | ab c.

Thus ca b | (a(ab c) (ca b)), that is, ca b | (a + 1)(a 1)b.


If a is even, then since ca b is a power of two and (a 1)(a + 1) is odd, we must
have ca b | b. Thus ca 2b < 2a, which yields the contradiction c < 2. Henceforth
we may assume that a is odd. Also a > b > c 2 implies a 4, and hence a 5.
If 4 | a 1, then since ca b is a power of two and a + 1 is divisible by 2 but not
by 4, we have

ca b | 2(a 1)b
ca b | 2(a 1)b 2(ab c) (ca b | ab c)
ca b | 2(b c)
ca b 2(b c). (b = c)

If 4 | a + 1, then since a 1 is divisible by 2 but not by 4, we have

ca b | 2(a + 1)b
ca b | 2(a + 1)b 2(ab c) (ca b | ab c)
ca b | 2(b + c)
ca b 2(b + c).

Thus whatever odd number a is, we can be sure that ca b 2(b + c). That is,

ca 3b + 2c.

Since b, c < a, we have ca < 3a + 2a = 5a, and so c < 5.


If c = 4, then 4a < 3a + 8. Thus a < 8. But a > b > c = 4 implies a 6. Thus
a = 7 because a is odd. But since cb a = 4b 7 is a power of two and it is odd,
we must have 4b 7 = 1. Thus b = 2, which contradicts b > c.
If c = 3, then 3a 3b + 6 < 3a + 6. Thus a b + 2 < a + 2.
If b = a 1, then ca b = 3a (a 1) = 2a + 1 cannot be a power of two because
it is odd and greater than 1.
If b = a 2, then bc a = 3(a 2) a = 2a 6 and ca b = 3a (a 2) = 2a + 2
are two powers of two that differ by 8. The only such powers of two are 8 and
16. This quickly yields a = 7 and implies (a, b, c) = (7, 5, 3).

63

90
If c = 2, then bc a = 2b a = 1 because it is odd (a is odd) and is a power of two.
But

ca b | ab c
3b 2 | (2b 1)b 2 (a = 2b 1, c = 2)
3b 2 | 3((2b 1)b 2) 2b(3b 2)
3b 2 | b 6.

Since 3b 2 > b 6 > 0 for b 7, we must have b 6.


Checking 3b 2 | b 6 directly for b = 3, 4, 5, 6, we find that only b = 6 works. This
implies (a, b, c) = (11, 6, 2). 

64

91
3. Solution 1 (Alex Gunning, year 12, Glen Waverley Secondary College, VIC. Alex
was a Gold medallist with the 2015 Australian IMO team.)
Let A be the point diametrically opposite A on and let E be the second point of
intersection of the line AHF with .
Lemma The points A , M , H and Q are collinear.
Proof First note that QA QA because AA is a diameter of . Also since
QA QH, it follows that QHA is a straight line. Thus Q lies on the line A H.

B F C

A E

Next we show that CHBA is a parallelogram.


Since AA is a diameter of we have AE A E. However AE BC. Thus
BC  A E. Since A EBC is cyclic, it follows that A ECB is an isosceles trapezium
with CE = A B. Thus

BCA = EBC (A B = CE)


= EAC (ECAB cyclic)
= 90 ACB (AE BC)
= CBH. (BH AC)

Thus BH  A C. A similar argument shows that CH  A B, and so CHBA is a


parallelogram.
Finally, since the diagonals of any parallelogram bisect each other, and M is the
midpoint of BC, it follows that M is also the midpoint of A H. Thus M lies on the
line A H. Since we already know that Q lies on the line A H, it follows that A , M ,
H and Q are collinear. 
Let R be the intersection of the line A E with the line through H that is perpendic-
ular to A Q. Observe that RH is tangent to circles KQH, F HM and EHA . This
is because each of these circles has a diameter which lies on the line A Q. Applying
the radical axis theorem to the three circles , KQH and EHA , we deduce that
lines RH, A E and QK are concurrent. Hence R lies on QK.

65

92
A

S
B M F C

A E R

Recall that M is the midpoint of A H and that BC  A E. Thus the midpoint, S


say, of RH lies on the line BC. Since HK KR we have that S is the centre of
circle RKH, and so SH = SK. But since SH is tangent to circle KQH at H and
SH = SK, it follows that SK is tangent to circle KQH at K. Considering the
power of point S with respect to circle F HM , we have

SF SM = SH 2 = SK 2 .

It follows that SK is tangent to circle F KM at K. Since circle KQH is also tangent


to SK at K, we conclude that circles KQH and F KM are tangent to each other
at K. 

66

93
Solution 2 (Andrew Elvey Price, Deputy Leader of the 2015 Australian IMO
team)
Point A is defined as in solution 1. Furthermore, as in solution 1, we establish that
M is the midpoint of A H, and A , M , H and Q are collinear.
Since A AK = A QK = HQK and AKA = 90 = QKH, we have
KAA KQH. Hence there is a spiral symmetry, f say, centred at K, such
that f (A ) = A and f (H) = Q. Note that f is the composition of a 90 rotation
KA
about K with a dilation of factor KA  about K.

Let M  and F  be the respective images of M and F under f . Thus M  F  M F .


But since AH M F , it follows that M  F   AH.
Since M is the midpoint of A H, it follows that M  is the midpoint of AQ. Since
M  F   AH, it follows that M  F  passes through the midpoint, S  say, of HQ. Note
that S  is the centre of circle KQH.

M

F Q

S

K
H

M
B F S C

A

Let S be the preimage of S  under f . (Note that S happens to lie on the line M F ,
as shown in the diagram, because S  lies on line M  F  . But we will not need this
fact.)
Since S  F  Q S  QM  (AA), we have

S F  S Q
= .
S Q S M 
It follows that
S  F  S  M  = S  Q2 = S  K 2 .
Therefore, S  K is tangent to circle F  KM  at K.
Applying the inverse of f , we have SK is tangent to circle F KM at K. But since
S is the preimage of S  under f , we also have S  KS = 90 , and so SK is tangent
to circle KQH. Hence circles F KM and KQH are tangent at K. 

67

94
Solution 3 (IMO Problem Selection Committee)
Points A and E are defined as in solution 1. Moreover, as in solution 1, we establish
that M is the midpoint of A H, and A , M , H and Q are collinear. Note that since
M F  A E, it follows that F is the midpoint of EH.
Let Q be the point diametrically opposite Q on . Then KQ KQ because QQ
is a diameter of . Also since KQ KH, it follows that KHQ is a straight line.
Let T be a point on the tangent to circle KQH at K, such that T and Q lie
on the same side of the line KH. By the alternate segment theorem we have
T KQ = KHQ.
By the alternate segment theorem, it is sufficient to prove that

KF M = T KM.

We have

KF M = T KM
90 + KF A = T KQ + 90 + HKM

KF A = Q HA + HKM. (1)

Thus it suffices to establish (1).


Let J be the midpoint of HQ . Observe that triangles KHE and AHQ are similar
with F and J being the midpoints of corresponding sides. Hence KF A = HJA.

Q T

H K
J
Q

B M F C

A E

Observe also that triangles KHA and QHQ are similar with M and J being the
midpoints of corresponding sides. Hence HKM = JQH.
Thus our task is reduced to proving

HJA = Q HA + JQH.

Let us draw a new diagram that will help us focus on the task at hand.

68

95
A

O
J H
Q

A

Note that QAQ A is a rectangle. Let O be its centre. We also know that H lies
on side A Q and that J is the midpoint of Q H. Thus J and O both lie on the
mid-parallel of QA and Q A. Hence

HJA = HJO + OJA


= Q HA + Q AJ. (A Q  JO  Q A)

Thus it suffices to prove that JQH = Q AJ. However, this is an immediate


consequence of the fact that JO a line of reflective symmetry of the rectangle. 

69

96
Solution 4 (Jacob Tsimerman, Leader of the 2015 Canadian IMO team)
Points A , E and Q are defined as in solution 3. Furthermore, as in solution 3, we
have M is the midpoint of A H; points A , M , H and Q are collinear; points Q , H
and K are collinear; and F is the midpoint of HE.
Since the three chords AE, QA and KQ are concurrent we have

HA HE = HQ HA = HK HQ = r2 ,

for some positive real number r. Let f be the inversion with centre H and radius
r, followed by the reflection11 in H. Let A and Q be the reflections of H in A and
Q, respectively. Then f has the effect of exchanging the following pairs of points.

AE A F Q A Q M K Q

A

A
Q

H K

Q

B M F C

A E

Hence circle KQH is transformed into line Q A , and circle F KM is transformed


into circle A Q Q . So it suffices to show that Q A is tangent to circle A Q Q . Since
Q A  AQ  A Q , it suffices to show that A Q Q is isosceles with Q A = Q Q .
Observe Q A  A Q because AQA Q is a rectangle. So Q A A Q . But A is the
centre of circle A HQ because it is the midpoint of A H and A Q H = 90 . Thus
Q A is the perpendicular bisector of A Q . From this it follows that Q A = A Q ,
as desired. 

11 Reflection in a point is the same as a 180 rotation about that point.

70

97
Comment Solutions 1 and 3 first establish that Q, H, M and A are collinear, and
that M and F are the respective midpoints of A H and EH. After this the points
B and C are no longer relevant to the solution. The crux of matter boils down to
the following.

Let AQA Q be a rectangle inscribed in a circle . Let H be any point on


the line A Q. Let E and K be the respective second points of intersection
of the lines AH and Q H with . Let M and F be the midpoints of A H
and EH, respectively. Then circles KQH and KM F are tangent to each
other.

H K

Q

M F

A E

71

98
Here is an even more minimal way of looking at the same thing.

Let A EKQ be a cyclic quadrilateral. Let H be a point on the line A Q


such that HEA = QKH = 90 . Let M and F be the midpoints of
A H and EH, respectively. Then circles KQH and KM F are tangent to
each other. (*)

K
H

M F

A E

72

99
Solution 5 (Panupong Pasupat, one of the Coordinators at the 2015 IMO)
It is sufficient to prove statement (*) found in the comments on the previous page.
Let T be a point on the tangent to circle KQH at K, such that T and Q lie on the
same side of the line KH. Let W be the midpoint of QH. Since QKH = 90 , it
follows that W is the centre of circle KQH. Hence W H = W K, and we may let
W KH = KHW = .

From the angle sum in KQH, we have HKQ = 90 . Since A EKQ is cyclic,
we have AEK = 90 + , and so KEH = . Thus by the alternate segment
theorem, circle HEK is tangent to line A Q at H. Then since M H = M E, it
follows by symmetry that M H and M E are the common tangents from M to circle
HEK.

Q T

K
H

M
F

A E

The configuration where M H and M E are common tangents to the circumcircle of


HEK, is a standard one. In particular it yields an alternative characterisation of
the symmedian.22 Thus we may let
HKM = F KE = .

From the exterior angle sum in EKF , we have


KF H = + = W KM. (1)

Since W K is the radius of circle KQH, we have T K KW . Thus adding 90 to


both sides of (1) yields
KF M = T KM.
Hence from the alternate segment theorem, T K is tangent to circle F KM at K.
Thus circles F KM and KQH are tangent at K. 
2
2
For more details see the section Alternative characterisations of symmedian found in chapter 5 of Problem
Solving Tactics published by the AMT.

73

100
4. Solution 1 (Kevin Xian, year 11, James Ruse Agricultural High School, NSW.
Kevin was a Silver medallist with the 2015 Australian IMO team.)

X
K
L
F

G
O

B D E C

Observe that AF = AG (radius of ), and OF = OG (radius of ). So F and


G are symmetric in AO. Thus lines F K and GL intersect on AO if and only if
KF A = AGL. We have,

KF A = F KB F AB (exterior angle AF K)


= F DB F GB (BDKF and AF BG cyclic)
= F GE F GB (F DEG cyclic)
= BGE
= CEG CBG (exterior angle GEB)
= CLG CAG (ECGL and ABCG cyclic)
= AGL. (exterior angle GLA) 

Comment A careful analysis of this solution shows that the result is still true if
we only assume that the centre of lies on the line AO.

74

101
Solution 2 (Found independently by Ilia Kucherov, year 11, Westall Secondary
College, VIC, and Jeremy Yip, year 12, Trinity Grammar School, VIC. Ilia and
Jeremy were Silver medallists with the 2015 Australian IMO team.)
As in solution 1 it suffices to show that KF A = AGL.
Let the lines DF and EG intersect for a second time at points P and Q, respec-
tively. Then

P DE = F GQ (F DEG cyclic)
= F P Q. (F QP G cyclic)

Thus BC  QP . Since BQP C is cyclic, it follows that BQP C is an isosceles


trapezium with BQ = P C. Hence BGQ = P F C, that is,

BGE = DF C. (1)

B D E C

Q P

75

102
A

X
K
L
F

G
O

B D E C

Since ABCG is cyclic, we have LAG = CAG = CBG = EBG. Since ECGL
is cyclic, CLG = CEG, and so GLA = GEB. Thus GAL GBE (AA).
Hence
AGL = BGE. (2)
Similarly
KF A = DF C. (3)

Combining (1), (2) and (3), it follows that

KF A = AGL. 

Comment There are two pairs of similar triangles associated with circles ECGL
and . They are GAL GBE and GLE GAB. This is a standard
configuration which can help fast track the route to a solution.33

3
3
For more details see the section Similar Switch found in chapter 5 of Problem Solving Tactics published
by the AMT.

76

103
5. Solution 1 (Seyoon Ragavan, year 11, Knox Grammar School, NSW. Seyoon was
a Gold medallist with the 2015 Australian IMO team.)
We show that the only answers are: f (x) = x for all x R and f (x) = 2 x for all
x R.
We are asked to find all functions f : R R such that for all x, y R,

f (x + f (x + y)) + f (xy) = x + f (x + y) + yf (x). (1)

Set y = 1 in (1) to find that for all x R,

x + f (x + 1) is a fixed point of f . (2)

With (2) in mind, set x = 0 and y = z + f (z + 1) in (1) to find that for all z R,

f (0) = f (0)(z + f (z + 1)). (3)

Case 1 f (0) = 0
Equation (3) implies z + f (z + 1) = 1 for all z R. Putting z = x 1, this may be
rearranged to f (x) = 2 x for all x R. We verify this is a solution.

LHS = 2 (x + 2 (x + y)) + 2 xy = 2 + y xy
RHS = x + 2 (x + y) + y(2 x) = 2 + y xy = LHS

Case 2 f (0) = 0
Set x = 0 in (1) to find that for all y R,

f (f (y)) = f (y). (4)

Set y = 0 in (1) to find that for all x R,

x + f (x) is a fixed point of f . (5)

Let S denote the set of fixed points of f . Suppose that u S. Then (5) with x = u
tells us that 2u S. And (2) with x = u 1 tells us that 2u 1 S. Hence

u S 2u, 2u 1 S. (6)

Since 0 S, applying (6) tells us that 1 S. Applying (6) again tells us that
2, 3 S. Continuing inductively, we find that all negative integers are in S.
On the other hand, if x is any positive integer, choose an integer y such that y < 0,
x + y < 0, 2x + y < 0 and xy < 0 (any y < 2x will do). Using these values in (1)
yields f (x) = x. Hence
Z S. (7)

Since f (1) = 1, we may set x = 1 in (1) to find that for all y R,

f (1 + f (y + 1)) + f (y) = y + 1 + f (y + 1). (8)

Suppose that u, u + 1 S. Then setting y = u in (8) tells us that u + 2 S. Also


setting y = u 1 in (8) tells us that u 1 S. An immediate corollary of this is

u, u + 1 S u + n S for any n Z. (9)

77

104
Let y R. Then from (2) with x = y 1 we have y + f (y) 1 S, and from (5) we
have y + f (y) S. Hence with u = y + f (y) 1 in (7), we find that for each n Z,

y + f (y) + n S. (10)

Using the change of variables y = x + m and n = m, we have for each m Z and


x R,
x + f (x + m) S. (11)

If we set y = m in (1) and use (11), then for each m Z and x R we have,

f (mx) = mf (x). (12)

If we replace y with f (y) in (10) and remember that f (f (y)) = f (y) from (4), we
find
2f (y) + n S. (13)

Let y R and let y = 2x. Then applying (12) and (13), we are able to deduce that
f (y) + 1 = f (2x) + 1 = 2f (x) + 1 S. Hence for all y R we have

f (y) + 1 S. (14)

Finally, put x = 1 in (1). Then using f (1 + f (1 + y)) = 1 + f (1 + y) from (14), and


f (1) = 1 from (7), we deduce that f (y) = y for all y R. This is easily seen to be
a solution. 

78

105
Solution 2 (Based on the presentation by Alex Gunning, year 12, Glen Waverley
Secondary College, VIC. Alex was a Gold medallist with the 2015 Australian IMO
team.)
We are asked to find all functions f : R R such that for all x, y R,

f (x + f (x + y)) + f (xy) = x + f (x + y) + yf (x). (1)

The case f (0) = 0 is handled as in solution 1.


It remains to deal with the case f (0) = 0. All equations that follow will hold true
for all z R.
Replacing x with y and y with x in (1) yields

f (y + f (x + y)) + f (xy) = y + f (x + y) + xf (y). (2)

Computing the difference (1) (2), we have for all x, y R,

f (x + f (x + y)) f (y + f (x + y)) = x y + yf (x) xf (y). (3)

Putting y = x in (3) yields

f (x) f (x) = 2x xf (x) xf (x). (4)

Suppose that f (x) = x. Then (4) can be rearranged to yield

(x 1)(f (x) + x)) = 0.

If x = 1, then f (x) = x. If x = 1, then putting (x, y) = (1, 1) in (1), yields


f (1) = 1. Hence if S is the set of fixed points of f , we have shown that

x S x S. (5)

Putting (x, y) = (z, 0) in (1), we find,

z + f (z) S. (6)

Putting (x, y) = (0, z) in (1), we find,

f (z) S. (7)

Putting (x, y) = (f (z), 0) in (1) and using (7), we find,

2f (z) S. (8)

Put (x, y) = (z + f (z), f (z)) in (3). The LHS of (3) is f (z + 2f (z)). We also
have x = z + f (z) S from (6). And y = f (z) S from (7) and (5). Thus
yf (x) xf (y) = yx xy = 0, and so the RHS of (3) is just x y = z + 2f (z).
Hence
z + 2f (z) S. (9)

79

106
Put (x, y) = (z, z f (z)) in (3). Note that y = z f (z) S from (6) and (5).
And x + y = f (z) S from (7) and (5). Thus (3) simplifies to

f (2x + y) f (x + 2y) = x y + yf (x) xy.

However, x + 2y = z 2f (z) S from (9) and (5). So (3) simplifies further to

f (2x + y) = 2x + y + yf (x) xy.

Writing x and y in term of z, and tidying up yields

f (z f (z)) = z f (z) + z 2 f (z)2 . (10)

Put (x, y) = (z f (z), f (z)) in (3). The LHS of (3) equals f (z) f (2f (z)) = f (z)
from (8). Hence

f (z) = x y + yf (x) xf (y)


= z 2f (z) + yf (x) xy (since y = f (z) S)
f (z) z = f (z)(f (x) x) (y = f (z))
= f (z)(z 2 f (z)2 ). (by (10) as x = z f (z))

This can be written in the form

(f (z) z)(f (z)2 + zf (z) + 1) = 0. (11)


 
z+ z 2 4 z z 2 4
Solving for f (z) in (11), we find f (z) z, 2
, 2
Since f (z) R,
.
we have f (z) = z for all |z| < 2. Hence (2, 2) S. Finally, from (8) we see that
z S implies 2z S. This allows us to deduce that (2k , 2k ) S for all positive
integers k. Hence S = R, and f (x) = x for all x R. 

80

107
6. This was the hardest problem of the 2015 IMO. Only 11 of the 577 contestants were
able to solve this problem completely.
The authors of this problem were Ross Atkins and Ivan Guo of Australia. Ross and
Ivan were Bronze medallists with the 2003 Australian IMO Team and Ivan was a
Gold medallist with the 2004 Australian IMO Team. The problem was inspired by a
notation for juggling (Ross is also a juggler) in which each ai represents the airtime
of a ball thrown at time i, and b is the total number of balls.
Solution 1 (Alex Gunning, year 12, Glen Waverley Secondary College, VIC. Alex
was a Gold medallist with the 2015 Australian IMO team.)
Let S be the set of positive integers which are not of the form n + an for some
positive integer n. Note that S is nonempty because 1  S. Let s1 < s2 < be
the elements of S listed in increasing order.
Lemma |S| 2015.
Proof Assume that |S| 2016. Choose n so that an + n s2016 . Since an 2015,
this implies that s1 , s2 , . . . , s2016 {1, 2, . . . , n + 2015}. However, the n numbers
1 + a1 , 2 + a2 , . . . , n + an are not equal to any si and are also members of the set
{1, 2, . . . , n + 2015}. Hence {1, 2, . . . , n + 2015} contains at least n + 2016 different
numbers, contradiction. 
We claim that if b = |S| and if N is larger than all members of S, then the inequality
posed in the problem statement is true.
n
Let n be any integer
 satisfying n N . We shall find bounds for j=1 (j + aj ) and
hence also for nj=1 (aj b). In what follows, let L be the following list of n + b
distinct positive integers.
1 + a1 , 2 + a2 , . . . , n + an , s1 , s2 , . . . , sb

For the lower bound,


n+bsince the n + b numbers in L are distinct, their sum is greater
than or equal to j=1 j. Hence we have
n
 b
 n+b

(j + aj ) + sj j
j=1 j=1 j=1
n
 n+b
 b

aj j sj
j=1 j=n+1 j=1

b(2n + b + 1)
= s
2
n
b2 + b
(aj b) s, (1)
j=1
2
b
where s = j=1 sj .
For the upper bound, observe that s1 , s2 , . . . , sb are b members belonging to the set
T = {1, 2, . . . , n + 1}. The remaining n + 1 b members of T must be of the form
j + aj where j n, and so are in L. The sum of these n + 1 b numbers is exactly
n+1
 b

j sb .
j=1 j=1

81

108
All together there are exactly n numbers of the form j + aj in L and so far we have
accounted for n + 1 b of them.
Consider the remaining b1 numbers of the form j +aj which are in L. When listed
in decreasing order, they can be no larger than
 n+2015, n+2014, . . . , n+2015b+2,
respectively. Hence their sum is at most b1 j=1 (n + 2016 j). Thus

n
 n+1
 b
 b1

(j + aj ) j sb + (n + 2016 j)
j=1 j=1 j=1 j=1
n
 (b 1)(2n + 4032 b)
aj n + 1 + s
j=1
2
n
 4033b b2 4030
(aj b) s. (2)
j=1
2

Summarising (1) and (2), we have established the following bounds for any n N .

 n
4033b b2 4030 b2 + b
s (aj b) s. (3)
2 j=1
2

Now let m, n be any two integers satisfying n > m N . Since also m N , (3) is
also satisfied if n is replaced by m. Thus
   
  n   n m
 
   
 (aj b) =  (aj b) (aj b)
   
j=m+1 j=1 j=1
 2   
 b +b 4033b b2 4030 

 s s 
2 2

= (b 1)(2015 b)
 2
(b 1) + (2015 b)
(AMGM)
2
= 10072 . 

82

109
Solution 2 (A juggling interpretation solution by the authors Ross Atkins and
Ivan Guo)
Suppose you are juggling several balls using only one hand. At the ith second, if a
ball lands in your hand, it is thrown up immediately. If no ball lands, you instead
reach for an unused ball (that is, a ball that has not been thrown yet) and throw it
up. In both cases, a ball is thrown so that it will stay in the air for ai seconds. The
condition ai + i = aj + j ensures that no two balls land at the same time.
Let b be the total number of balls used. If b > 2015, then eventually a ball must
stay in the air for more than 2015 seconds, contradicting ai 2015. So b is finite
and bounded by 2015.
Select N so that all b balls have been introduced by the N th second. For all i N ,
denote by Ti the total remaining airtime of the current balls, immediately after the
ith throw is made. (That is, we calculate the remaining airtime for each current
ball, and add these values together.) Consider what happens during the next second.
The airtime of each of the b balls is reduced by 1. At the same time a ball is thrown,
increasing its airtime by ai+1 . Thus we have the equality Ti+1 Ti = ai+1 b. This
gives a nice representation of the required sum,
n

(ai b) = Tn Tm .
i=m+1

To complete the problem, it suffices to identify the maximal and minimal possible
values of the total remaining airtime Ti . Since no two balls can land at the same
time, the minimal value is 1 + 2 + + b. On the other hand, the maximal value
is 1 + 2015 + 2014 + + (2015 b + 2). (Note that there must be a ball with a
remaining airtime of 1 since something must be caught and thrown every second.)
Taking the difference between these two sums, we find that

(4032 b)(b 1) (b + 2)(b 1)


|Tn Tm |
2 2
= (2015 b)(b 1)
10072 (GM AM)

as required. 

83

110
INTERNATIONAL MATHEMATICAL OLYMPIAD
RESULTS
Mark distribution by question

Mark Q1 Q2 Q3 Q4 Q5 Q6

0 93 256 408 91 153 521

1 89 151 122 36 255 11

2 5 77 12 61 34 15

3 21 27 1 18 90 6

4 72 8 3 11 8 3

5 12 13 0 1 4 3

6 20 14 1 8 3 7

7 265 31 30 351 30 11

Total 577 577 577 577 577 577

Mean 4.3 1.4 0.7 4.8 1.5 0.4

Australian scores at the IMO

Name Q1 Q2 Q3 Q4 Q5 Q6 Score Award

Alex Gunning 7 6 7 7 2 7 36 Gold

Ilia Kucherov 7 2 0 7 3 0 19 Silver

Seyoon Ragavan 7 7 1 7 7 0 29 Gold

Yang Song 7 2 1 7 3 0 20 Silver

Kevin Xian 7 3 1 7 3 0 21 Silver

Jeremy Yip 7 6 1 7 2 0 23 Silver

Totals 42 26 11 42 20 7 148

Australian average 7.0 4.3 1.8 7.0 3.3 1.2 24.7

IMO average 4.3 1.4 0.7 4.8 1.5 0.4 13.0

The medal cuts were set at 26 for Gold, 19 for Silver and 14 for Bronze.

111
Some country totals

Rank Country Total


1 United States of America 185
2 China 181
3 South Korea 161
4 North Korea 156
5 Vietnam 151
6 Australia 148
7 Iran 145
8 Russia 141
9 Canada 140
10 Singapore 139
11 Ukraine 135
12 Thailand 134
13 Romania 132
14 France 120
15 Croatia 119
16 Peru 118
17 Poland 117
18 Taiwan 115
19 Mexico 114
20 Hungary 113
20 Turkey 113
22 Brazil 109
22 Japan 109
22 United Kingdom 109
25 Kazakhstan 105
26 Armenia 104
27 Germany 102
28 Hong Kong 101
29 Bulgaria 100
29 Indonesia 100
29 Italy 100
29 Serbia 100

112
Distribution of awards at the 2015 IMO

Country Total Gold Silver Bronze HM


Albania 37 0 0 0 3
Algeria 60 0 1 1 2
Argentina 70 0 0 1 4
Armenia 104 0 1 5 0
Australia 148 2 4 0 0
Austria 63 0 0 3 1
Azerbaijan 73 0 0 2 4
Bangladesh 97 0 1 4 1
Belarus 84 0 0 3 3
Belgium 67 0 1 0 3
Bolivia 5 0 0 0 0
Bosnia and Herzegovina 76 0 0 2 4
Botswana 1 0 0 0 0
Brazil 109 0 3 3 0
Bulgaria 100 0 2 1 2
Cambodia 24 0 0 0 2
Canada 140 2 0 4 0
Chile 12 0 0 0 1
China 181 4 2 0 0
Colombia 72 0 0 4 0
Costa Rica 53 0 0 2 2
Croatia 119 1 3 1 0
Cuba 15 0 0 1 0
Cyprus 58 0 1 0 2
Czech Republic 74 0 0 3 3
Denmark 52 0 0 2 1
Ecuador 27 0 0 0 2
El Salvador 14 0 0 0 0
Estonia 51 0 0 1 3
Finland 26 0 0 0 1
France 120 0 3 3 0
Georgia 80 0 1 3 1
Germany 102 0 2 3 0
Ghana 5 0 0 0 0
Greece 71 0 1 2 2

113
Country Total Gold Silver Bronze HM
Hong Kong 101 0 2 3 1
Hungary 113 0 3 3 0
Iceland 41 0 0 0 3
India 86 0 1 2 3
Indonesia 100 0 2 4 0
Iran 145 3 2 1 0
Ireland 37 0 0 0 3
Israel 83 1 0 2 2
Italy 100 1 2 0 0
Japan 109 0 3 3 0
Kazakhstan 105 1 1 2 2
Kosovo 24 0 0 0 1
Kyrgyzstan 17 0 0 0 0
Latvia 36 0 0 0 3
Liechtenstein 18 0 0 1 0
Lithuania 54 0 0 1 1
Luxembourg 12 0 0 0 1
Macau 88 0 1 2 3
Macedonia (FYR) 45 0 0 1 1
Malaysia 66 0 0 3 1
Mexico 114 1 2 3 0
Moldova 85 0 1 2 3
Mongolia 74 0 0 2 4
Montenegro 19 0 0 1 0
Morocco 27 0 0 0 1
Netherlands 76 0 0 3 1
New Zealand 72 0 0 2 4
Nicaragua 26 0 0 0 3
Nigeria 22 0 0 0 2
North Korea 156 3 3 0 0
Norway 54 0 1 0 2
Pakistan 25 0 0 1 0
Panama 9 0 0 0 0
Paraguay 53 0 0 3 0
Peru 118 2 2 1 0
Philippines 87 0 2 2 1
Poland 117 1 1 4 0

114
Country Total Gold Silver Bronze HM
Portugal 70 0 0 3 1
Puerto Rico 18 0 0 1 0
Romania 132 1 4 1 0
Russia 141 0 6 0 0
Saudi Arabia 81 0 1 3 2
Serbia 100 1 1 2 2
Singapore 139 1 4 1 0
Slovakia 97 0 2 3 0
Slovenia 46 0 0 1 1
South Africa 68 0 0 1 2
South Korea 161 3 1 2 0
Spain 47 0 0 1 2
Sri Lanka 51 0 0 0 4
Sweden 63 0 0 2 2
Switzerland 74 0 0 3 2
Syria 69 0 1 1 3
Taiwan 115 0 4 1 1
Tajikistan 57 0 1 1 2
Tanzania 0 0 0 0 0
Thailand 134 2 3 1 0
Trinidad and Tobago 26 0 1 0 0
Tunisia 41 0 0 1 2
Turkey 113 0 5 0 0
Turkmenistan 64 0 0 2 2
Uganda 6 0 0 0 0
Ukraine 135 2 3 1 0
United Kingdom 109 0 4 1 1
United States of America 185 5 1 0 0
Uruguay 16 0 0 0 1
Uzbekistan 64 0 0 3 2
Venezuela 13 0 0 0 1
Vietnam 151 2 3 1 0
Total (104 teams, 577 contestants) 39 100 143 126

115
ORIGIN OF SOME QUESTIONS
ORIGIN OF SOME QUESTIONS
Senior Contest
Question 1 was submitted by Angelo Di Pasquale.
Questions 2, 3 and 5 were submitted by Norman Do.
Question 4 was submitted by Alan Offer.

Australian Mathematical Olympiad


Questions 1, 2, 5 and 6 were submitted by Norman Do.
Question 3 was submitted by Andrei Storozhev.
Questions 4 and 7 were submitted by Angelo Di Pasquale.
Question 8 was submitted by Andrew Elvey Price.

Asian Pacific Mathematical Olympiad 2015


Question 2 was composed by Angelo Di Pasquale and submitted by the AMOC Senior Problems Com-
mittee.
International Mathematical Olympiad 2015
Question 6 was composed by Ross Atkins and Ivan Guo, and submitted by the AMOC Senior Problems
Committee. Ivan provided the following background information on the problem.

The original idea for this problem came about while Ross was reading the paper Positroid
Varieties: Juggling and Geometry by Knutson, Lam and Speyer, in which the excitation
number of a periodic juggling sequence was discovered. It seemed obvious that this was
similar to some specific elementary result that could be proven using elementary methods.
We had some difficulties in phrasing the problem in a concise self-contained way. Intuitively,
each term ai in the sequence corresponds to throwing a ball at the ith second with an air time
of ai . The inequality condition ensures that no two balls land simultaneously.
The first formulation of the problem was to show that the long-term average of the sequence
converges to an integer b, which is the total number of balls. However, the usage of limits was
inappropriate for an olympiad problem. We then came up with three more versions which
involved bounding the partial
 sums. Eventually we settled on the most difficult version, with
the explicit bound of | (ai b)| 10072 . Interestingly, the term ai b can be interpreted
as the change in the total air time on the ith second, while 10072 is the difference between
maximal and minimal possible total air times, after the introduction of all b balls. The final
wording may be a little difficult for students who are unfamiliar with the construct: there
exists an N such that for all m > n > N .
It is possible to solve the problem combinatorially without invoking any physical interpreta-
tions, juggling or otherwise. Furthermore, as demonstrated by some at the IMO, the problem
can also be tackled using purely algebraic approaches. Overall, we are very happy with the
problem and we hope everyone enjoyed it.

It is worth noting that one of the authors of the paper that inspired this problem was Thomas Lam, a
member of the 1997 Australian IMO team and recipient of an IMO gold medal.

116
HONOUR ROLL

Because of changing titles and affiliations, the most senior title achieved and later affiliations are generally used,
except for the Interim committee, where they are listed as they were at the time.

Interim Committee 19791980


Mr P J OHalloran Canberra College of Advanced Education, ACT, Chair
Prof A L Blakers University of Western Australia
Dr J M Gani Australian Mathematical Society, ACT,
Prof B H Neumann Australian National University, ACT,
Prof G E Wall University of Sydney, NSW
Mr J L Williams University of Sydney, NSW

Australian Mathematical Olympiad Committee


The Australian Mathematical Olympiad Committee was founded at a meeting of the Australian Academy of
Science at its meeting of 23 April 1980.
* denotes Executive Position

Chair*
Prof B H Neumann Australian National University, ACT 7 years; 19801986
Prof G B Preston Monash University, VIC 10 years; 19861995
Prof A P Street University of Queensland 6 years; 19962001
Prof C Praeger University of Western Australia 14 years; 20022015
Deputy Chair*
Prof P J OHalloran University of Canberra, ACT 15 years; 19801994
Prof A P Street University of Queensland 1 year; 1995
Prof C Praeger, University of Western Australia 6 years; 19962001
Assoc Prof D Hunt University of New South Wales 14 years; 20022015
Executive Director*
Prof P J OHalloran University of Canberra, ACT 15 years; 19801994
Prof P J Taylor University of Canberra, ACT 18 years; 19942012
Adj Prof M G Clapper University of Canberra, ACT 3 years; 20132015

Secretary
Prof J C Burns Australian Defence Force Academy, ACT 9 years; 19801988
Vacant 4 years; 19891992
Mrs K Doolan Victorian Chamber of Mines, VIC 6 years; 19931998

Treasurer*
Prof J C Burns Australian Defence Force Academy, ACT 8 years; 19811988
Prof P J OHalloran University of Canberra, ACT 2 years; 19891990
Ms J Downes CPA 5 years; 19911995
Dr P Edwards Monash University, VIC 8 years; 19952002
Prof M Newman Australian National University, ACT 6 years; 20032008
Dr P Swedosh The King David School, VIC 7 years; 20092015

Director of Mathematics Challenge for Young Australians*


Mr J B Henry Deakin University, VIC 17 years; 19902006
Dr K McAvaney Deakin University, VIC 10 years; 20062015

Chair, Senior Problems Committee


Prof B C Rennie James Cook University, QLD 1 year; 1980
Mr J L Williams University of Sydney, NSW 6 years; 19811986
Assoc Prof H Lausch Monash University, VIC 27 years; 19872013
Dr N Do Monash University, VIC 2 years; 20142015
117
Director of Training*
Mr J L Williams University of Sydney, NSW 7 years; 19801986
Mr G Ball University of Sydney, NSW 3 years; 19871989
Dr D Paget University of Tasmania 6 years; 19901995
Dr M Evans Scotch College, VIC 3 months; 1995
Assoc Prof D Hunt University of New South Wales 5 years; 19962000
Dr A Di Pasquale University of Melbourne, VIC 15 years; 20012015

Team Leader
Mr J L Williams University of Sydney, NSW 5 years; 19811985
Assoc Prof D Hunt University of New South Wales 9 years; 1986, 1989, 1990, 19962001
Dr E Strzelecki Monash University, VIC 2 years; 1987, 1988
Dr D Paget University of Tasmania 5 years; 19911995
Dr A Di Pasquale University of Melbourne, VIC 13 years; 20022010, 20122015
Dr I Guo University of New South Wales 1 year; 2011

Deputy Team Leader


Prof G Szekeres University of New South Wales 2 years; 19811982
Mr G Ball University of Sydney, NSW 7 years; 19831989
Dr D Paget University of Tasmania 1 year; 1990
Dr J Graham University of Sydney, NSW 3 years; 19911993
Dr M Evans Scotch College, VIC 3 years; 19941996
Dr A Di Pasquale University of Melbourne, VIC 5 years; 19972001
Dr D Mathews University of Melbourne, VIC 3 years; 20022004
Dr N Do University of Melbourne, VIC 4 years; 20052008
Dr I Guo University of New South Wales 4 years; 200910, 20122013
Mr G White University of Sydney, NSW 1 year; 2011
Mr A Elvey Price Melbourne University, VIC 2 years; 20142015

State Directors
Australian Capital Territory
Prof M Newman Australian National University 1 year; 1980
Mr D Thorpe ACT Department of Education 2 years; 19811982
Dr R A Bryce Australian National University 7 years; 19831989
Mr R Welsh Canberra Grammar School 1 year; 1990
Mrs J Kain Canberra Grammar School 5 years; 19911995
Mr J Carty ACT Department of Education 17 years; 19952011
Mr J Hassall Burgmann Anglican School 2 years; 20122013
Dr C Wetherell Radford College 2 years; 20142015
New South Wales
Dr M Hirschhorn University of New South Wales 1 year; 1980
Mr G Ball University of Sydney, NSW 16 years; 19811996
Dr W Palmer University of Sydney, NSW 19 years; 19972015
Northern Territory
Dr I Roberts Charles Darwin University 2 years; 20142015
Queensland
Dr N H Williams University of Queensland 21 years; 19802000
Dr G Carter Queensland University of Technology 10 years; 20012010
Dr V Scharaschkin University of Queensland 4 years; 20112014
Dr A Offer Queensland 1 year; 2015
South Australia/Northern Territory
Mr K Hamann SA Department of Education 19 years; 19801982, 19912005, 2013
Mr V Treilibs SA Department of Education 8 years; 19831990
Dr M Peake Adelaide 8 years; 20062013

118
Dr D Martin Adelaide 2 years; 20142015
Tasmania
Mr J Kelly Tasmanian Department of Education 8 years; 19801987
Dr D Paget University of Tasmania 8 years; 19881995
Mr W Evers St Michaels Collegiate School 9 years; 19952003
Dr K Dharmadasa University of Tasmania 12 years; 20042015
Victoria
Dr D Holton University of Melbourne 3 years; 19801982
Mr B Harridge Melbourne High School 1 year; 1982
Ms J Downes CPA 6 years; 19831988
Mr L Doolan Melbourne Grammar School 9 years; 19891998
Dr P Swedosh The King David School 18 years; 19982015
Western Australia
Dr N Hoffman WA Department of Education 3 years; 19801982
Assoc Prof P Schultz University of Western Australia 14 years; 19831988, 19911994, 19961999
Assoc Prof W Bloom Murdoch University 2 years; 19891990
Dr E Stoyanova WA Department of Education 7 years; 1995, 20002005
Dr G Gamble University of Western Australia 10 years; 20062015

Editor
Prof P J OHalloran University of Canberra, ACT 1 year; 1983
Dr A W Plank University of Southern Queensland 11 years; 19841994
Dr A Storozhev Australian Mathematics Trust, ACT 15 years; 19942008
Editorial Consultant
Dr O Yevdokimov University of Southern Queensland 7 years; 20092015

Other Members of AMOC (showing organisations represented where applicable)


Mr W J Atkins Australian Mathematics Foundation 18 years; 19952012
Dr S Britton University of Sydney, NSW 8 years; 19901998
Prof G Brown Australian Academy of Science, ACT 10 years; 1980, 19861994
Dr R A Bryce Australian Mathematical Society, ACT 10 years; 19911998
Mathematics Challenge for Young Australians 13 years; 19992012
Mr G Cristofani Department of Education and Training 2 years; 19931994
Ms L Davis IBM Australia 4 years; 19911994
Dr W Franzsen Australian Catholic University, ACT 9 years; 19901998
Dr J Gani Australian Mathematical Society, ACT 1980
Assoc Prof T Gagen ANU AAMT Summer School 6 years; 19931998
Ms P Gould Department of Education and Training 2 years; 19951996
Prof G M Kelly University of Sydney, NSW 6 years; 19821987
Prof R B Mitchell University of Canberra, ACT 5 years; 19911995
Ms Anna Nakos Mathematics Challenge for Young Australians 13 years; 20032015
Mr S Neal Department of Education and Training 4 years; 19901993
Prof M Newman Australian National University, ACT 15 years; 19861998
Mathematics Challenge for Young Australians 10 years; 19992002,
(Treasurer during the interim) 20092014
Prof R B Potts University of Adelaide, SA 1 year; 1980
Mr H Reeves Australian Association of Maths Teachers 11 years; 19881998
Australian Mathematics Foundation 20142015
Mr N Reid IBM Australia 3 years; 19881990
Mr R Smith Telecom Australia 5 years; 19901994
Prof P J Taylor Australian Mathematics Foundation 6 years; 19901994, 2013
Prof N S Trudinger Australian Mathematical Society, ACT 3 years; 19861988
Assoc Prof I F Vivian University of Canberra, ACT 1 year; 1990
Dr M W White IBM Australia 9 years; 19801988

119
Associate Membership (inaugurated in 2000)
Ms S Britton 16 years; 20002015
Dr M Evans 16 years; 20002015
Dr W Franzsen 16 years; 20002015
Prof T Gagen 16 years; 20002015
Mr H Reeves 16 years; 20002015
Mr G Ball 16 years; 20002015

AMOC Senior Problems Committee


Current members
Dr N Do Monash University, VIC (Chair) 2 years; 20142015
(member) 11 years; 20032013
M Clapper Australian Mathematics Trust 3 years; 2013-2015
Dr A Di Pasquale University of Melbourne, VIC 15 years; 20012015
Dr M Evans Australian Mathematical Sciences Institute, VIC 26 years; 19902015
Dr I Guo University of Sydney, NSW 8 years; 20082015
Dr J Kupka Monash University, VIC 13 years; 20032015
Dr K McAvaney Deakin University, VIC 20 years; 19962015
Dr D Mathews Monash University, VIC 15 years; 20012015
Dr A Offer Queensland 4 years; 20122015
Dr C Rao IBM Australia 16 years; 20002015
Dr B B Saad Monash University, VIC 22 years; 19942015
Dr J Simpson Curtin University, WA 17 years; 19992015
Dr I Wanless Monash University, VIC 16 years; 20002015
Previous members
Mr G Ball University of Sydney, NSW 16 years; 19821997
Mr M Brazil LaTrobe University, VIC 5 years; 19901994
Dr M S Brooks University of Canberra, ACT 8 years; 19831990
Dr G Carter Queensland University of Technology 10 years; 20012010
Dr J Graham University of Sydney, NSW 1 year; 1992
Dr M Herzberg Telecom Australia 1 year; 1990
Assoc Prof D Hunt University of New South Wales 29 years; 19862014
Dr L Kovacs Australian National University, ACT 5 years; 19811985
Assoc Prof H Lausch Monash University, VIC (Chair) 27 years; 19872013
(member) 2 years; 2014-2015
Dr D Paget University of Tasmania 7 years; 19891995
Prof P Schultz University of Western Australia 8 years; 19932000
Dr L Stoyanov University of Western Australia 5 years; 20012005
Dr E Strzelecki Monash University, VIC 5 years; 19861990
Dr E Szekeres University of New South Wales 7 years; 19811987
Prof G Szekeres University of New South Wales 7 years; 19811987
Em Prof P J Taylor Australian Capital Territory 1 year; 2013
Dr N H Williams University of Queensland 20 years; 19812000

Mathematics School of Excellence


Dr S Britton University of Sydney, NSW (Coordinator) 2 years; 19901991
Mr L Doolan Melbourne Grammar, VIC (Coordinator) 6 years; 1992, 19931997
Mr W Franzsen Australian Catholic University, ACT (Coordinator) 2 years; 19901991
Dr D Paget University of Tasmania (Director) 5 years; 19901994
Dr M Evans Scotch College, VIC 1 year; 1995
Assoc Prof D Hunt University of New South Wales (Director) 4 years; 19961999
Dr A Di Pasquale University of Melbourne, VIC (Director) 16 years; 20002015

120
International Mathematical Olympiad Selection School
Mr J L Williams University of Sydney, NSW (Director) 2 years; 19821983
Mr G Ball University of Sydney, NSW (Director) 6 years; 19841989
Mr L Doolan Melbourne Grammar, VIC (Coordinator) 3 years; 19891991
Dr S Britton University of Sydney, NSW (Coordinator) 7 years; 19921998
Mr W Franzsen Australian Catholic University, ACT (Coordinator) 8 years; 19921996, 19992001
Dr D Paget University of Tasmania (Director) 6 years; 19901995
Assoc Prof D Hunt University of New South Wales (Director) 5 years; 19962000
Dr A Di Pasquale University of Melbourne, VIC (Director) 15 years; 20012015

121

You might also like